MedSerg Final Flashcards

1
Q
  1. A patient in her late fifties has expressed to the nurse her desire to explore hormone replacement therapy (HRT). Based on what aspect of the patients health history is HRT contraindicated?

A) History of vaginal dryness
B) History of hot flashes and night sweats
C) History of vascular thrombosis
D) Family history of osteoporosis

A

C) History of vascular thrombosis

How well did you know this?
1
Not at all
2
3
4
5
Perfectly
2
Q
  1. The nurse is administering eye drops to a patient with glaucoma. After instilling the patients first medication, how long should the nurse wait before instilling the patients second medication into the same eye?

A) 30 seconds
B) 1 minute
C) 3 minutes
D) 5 minutes

A

D) 5 minutes

How well did you know this?
1
Not at all
2
3
4
5
Perfectly
3
Q
  1. A nurse is performing a shift assessment on an elderly patient who is recovering after surgery for a hip fracture. The nurse notes that the patient is complaining of chest pain, has an increased heart rate, and increased respiratory rate. The nurse further notes that the patient is febrile and hypoxic, coughing, and producing large amounts of thick, white sputum. The nurse recognizes that this is a medical emergency and calls for assistance, recognizing that this patient is likely demonstrating symptoms of what complication?

A) Avascular necrosis of bone
B) Compartment syndrome
C) Fat embolism syndrome
D) Complex regional pain syndrome

A

C) Fat embolism syndrome

How well did you know this?
1
Not at all
2
3
4
5
Perfectly
4
Q
  1. The nurse is administering total parenteral nutrition (TPN) to a client who underwent surgery for gastric cancer. Which of the nurses assessments most directly addresses a major complication of TPN?

A) Checking the patients capillary blood glucose levels regularly
B) Having the patient frequently rate his or her hunger on a 10-point scale
C) Measuring the patients heart rhythm at least every 6 hours
D) Monitoring the patients level of consciousness each shift

A

A) Checking the patients capillary blood glucose levels regularly

How well did you know this?
1
Not at all
2
3
4
5
Perfectly
5
Q
  1. A student nurse is doing clinical hours at an OB/GYN clinic. The student is helping to develop a plan of care for a patient with gonorrhea who has presented at the clinic. The student should include which of the following in the care plan for this patient?

A) The patient may benefit from oral contraceptives.
B) The patient must avoid use of tampons.
C) The patient is susceptible to urinary incontinence.
D) The patient should also be treated for chlamydia.

A

D) The patient should also be treated for chlamydia.

How well did you know this?
1
Not at all
2
3
4
5
Perfectly
6
Q
6.	A nurse is providing care for a patient who has a recent diagnosis of Pagets disease. When planning this patients nursing care, interventions should address what nursing diagnoses? Select all that apply.
A)	Impaired Physical Mobility
B)	Acute Pain
C)	Disturbed Auditory Sensory Perception
D)	Risk for Injury
E)	Risk for Unstable Blood Glucose
A

A) Impaired Physical Mobility
B) Acute Pain
C) Disturbed Auditory Sensory Perception
D) Risk for Injury

How well did you know this?
1
Not at all
2
3
4
5
Perfectly
7
Q
  1. A nurse is caring for a newly admitted patient with a suspected GI bleed. The nurse assesses the patients stool after a bowel movement and notes it to be a tarry-black color. This finding is suggestive of bleeding from what location?

A) Sigmoid colon
B) Upper GI tract
C) Large intestine
D) Anus or rectum

A

B) Upper GI tract

How well did you know this?
1
Not at all
2
3
4
5
Perfectly
8
Q
  1. The nurse is assessing a 53-year-old woman who has been experiencing dysmenorrhea. What questions should the nurse include in an assessment of the patients menstrual history? Select all that apply.

A) Do you ever experience bleeding after intercourse?
B) How long is your typical cycle?
C) Did you have any sexually transmitted infections in early adulthood?
D) When did your mother and sisters get their first periods?
E) Do you experience cramps or pain during your cycle?

A

A) Do you ever experience bleeding after intercourse?
B) How long is your typical cycle?
E) Do you experience cramps or pain during your cycle?

How well did you know this?
1
Not at all
2
3
4
5
Perfectly
9
Q
  1. A patient with a diagnosis of esophageal varices has undergone endoscopy to gauge the progression of this complication of liver disease. Following the completion of this diagnostic test, what nursing intervention should the nurse perform?

A) Keep patient NPO until the results of test are known.
B) Keep patient NPO until the patients gag reflex returns.
C) Administer analgesia until post-procedure tenderness is relieved.
D) Give the patient a cold beverage to promote swallowing ability.

A

B) Keep patient NPO until the patients gag reflex returns.

How well did you know this?
1
Not at all
2
3
4
5
Perfectly
10
Q
  1. A patients neck dissection surgery resulted in damage to the patients superior laryngeal nerve. What area of assessment should the nurse consequently prioritize?

A) The patients swallowing ability
B) The patients ability to speak
C) The patients management of secretions
D) The patients airway patency

A

A) The patients swallowing ability

How well did you know this?
1
Not at all
2
3
4
5
Perfectly
11
Q
  1. A 42 year-old patient tells the nurse that she has found a painless lump in her right breast during her monthly self-examination. She says that she is afraid that she has cancer. Which assessment finding would most strongly suggest that this patients lump is cancerous?

A) Eversion of the right nipple and mobile mass
B) A nonmobile mass with irregular edges
C) A mobile mass that is soft and easily delineated
D) Nonpalpable right axillary lymph nodes

A

B) A nonmobile mass with irregular edges

How well did you know this?
1
Not at all
2
3
4
5
Perfectly
12
Q
  1. A nurse is caring for a patient in the late stages of esophageal cancer. The nurse should plan to prevent or address what characteristics of this stage of the disease? Select all that apply.
A)	Perforation into the mediastinum
B)	Development of an esophageal lesion
C)	Erosion into the great vessels
D)	Painful swallowing
E)	Obstruction of the esophagus
A

A) Perforation into the mediastinum
C) Erosion into the great vessels
E) Obstruction of the esophagus

How well did you know this?
1
Not at all
2
3
4
5
Perfectly
13
Q
  1. A female patient has been achieving significant improvements in her ADLs since beginning rehabilitation from the effects of a brain hemorrhage. The nurse must observe and assess the patients ability to perform ADLs to determine the patients level of independence in self-care and her need for nursing intervention. Which of the following additional considerations should the nurse prioritize?

A) Liaising with the patients insurer to describe the patients successes.
B) Teaching the patient about the pathophysiology of her functional deficits.
C) Eliciting ways to get the patient to express a positive attitude.
D) Appraising the family’s involvement in the patients ADLs.

A

D) Appraising the family’s involvement in the patients ADLs.

How well did you know this?
1
Not at all
2
3
4
5
Perfectly
14
Q
  1. The results of a nurses musculoskeletal examination show an increase in the lumbar curvature of the spine. The nurse should recognize the presence of what health problem?

A) Osteoporosis
B) Kyphosis
C) Lordosis
D) Scoliosis

A

C) Lordosis

How well did you know this?
1
Not at all
2
3
4
5
Perfectly
15
Q
  1. A 37-year-old male patient presents at the emergency department (ED) complaining of nausea and vomiting and severe abdominal pain. The patients abdomen is rigid, and there is bruising to the patients flank. The patients wife states that he was on a drinking binge for the past 2 days. The ED nurse should assist in assessing the patient for what health problem?

A) Severe pancreatitis with possible peritonitis
B) Acute cholecystitis
C) Chronic pancreatitis
D) Acute appendicitis with possible perforation

A

A) Severe pancreatitis with possible peritonitis

How well did you know this?
1
Not at all
2
3
4
5
Perfectly
16
Q
  1. A 23-year-old woman comes to the free clinic stating I think I have a lump in my breast. Do I have cancer? The nurse instructs the patient that a diagnosis of breast cancer is confirmed by what?

A) Supervised breast self-examination
B) Mammography
C) Fine-needle aspiration
D) Chest x-ray

A

C) Fine-needle aspiration

How well did you know this?
1
Not at all
2
3
4
5
Perfectly
17
Q
  1. The nurse is caring for a patient in acute kidney injury. Which of the following complications would most clearly warrant the administration of polystyrene sulfonate (Kayexalate)?

A) Hypernatremia
B) Hypomagnesemia
C) Hyperkalemia
D) Hypercalcemia

A

C) Hyperkalemia

How well did you know this?
1
Not at all
2
3
4
5
Perfectly
18
Q
  1. An elderly patient has come in to the clinic for her twice-yearly physical. The patient tells the nurse that she is generally enjoying good health, but that she has been having occasional episodes of constipation over the past 6 months. What intervention should the nurse first suggest?

A) Reduce the amount of stress she currently experiences.
B) Increase carbohydrate intake and reduce protein intake.
C) Take herbal laxatives, such as senna, each night at bedtime.
D) Increase daily intake of water.

A

D) Increase daily intake of water.

How well did you know this?
1
Not at all
2
3
4
5
Perfectly
19
Q
  1. The nurse is teaching a patient preventative measures regarding vaginal infections. The nurse should include which of the following as an important risk factor?

A) High estrogen levels
B) Late menarche
C) Nonpregnant state
D) Frequent douching

A

D) Frequent douching

How well did you know this?
1
Not at all
2
3
4
5
Perfectly
20
Q
  1. The case manager is working with an 84-year-old patient newly admitted to a rehabilitation facility. When developing a care plan for this older adult, which factors should the nurse identify as positive attributes that benefit coping in this age group? Select all that apply.
A)	Decreased risk taking
B)	Effective adaptation skills
C)	Avoiding participation in untested roles
D)	Increased life experience
E)	Resiliency during change
A

B) Effective adaptation skills
D) Increased life experience
E) Resiliency during change

How well did you know this?
1
Not at all
2
3
4
5
Perfectly
21
Q
  1. A patient with type 1 diabetes has told the nurse that his most recent urine test for ketones was positive. What is the nurses most plausible conclusion based on this assessment finding?

A) The patient should withhold his next scheduled dose of insulin.
B) The patient should promptly eat some protein and carbohydrates.
C) The patients insulin levels are inadequate.
D) The patient would benefit from a dose of metformin (Glucophage).

A

C) The patients insulin levels are inadequate.

How well did you know this?
1
Not at all
2
3
4
5
Perfectly
22
Q
  1. A patient has sought care because of recent dark-colored stools. As a result, a fecal occult blood test has been ordered. The nurse should instruct the patient to avoid which of the following prior to collecting a stool sample?

A) NSAIDs
B) Acetaminophen
C) OTC vitamin D supplements
D) Fiber supplements

A

A) NSAIDs

How well did you know this?
1
Not at all
2
3
4
5
Perfectly
23
Q
  1. A triage nurse in the emergency department is assessing a patient who presented with complaints of general malaise. Assessment reveals the presence of jaundice and increased abdominal girth. What assessment question best addresses the possible etiology of this patients presentation?

A) How many alcoholic drinks do you typically consume in a week?
B) To the best of your knowledge, are your immunizations up to date?
C) Have you ever worked in an occupation where you might have been exposed to toxins?
D) Has anyone in your family ever experienced symptoms similar to yours?

A

A) How many alcoholic drinks do you typically consume in a week?

How well did you know this?
1
Not at all
2
3
4
5
Perfectly
24
Q
  1. An elderly patient is brought to the emergency department with a fractured tibia. The patient appears malnourished, and the nurse is concerned about the patients healing process related to insufficient protein levels. What laboratory finding would the floor nurse prioritize when assessing for protein deficiency?

A) Hemoglobin
B) Bilirubin
C) Albumin
D) Cortisol

A

C) Albumin

How well did you know this?
1
Not at all
2
3
4
5
Perfectly
25
Q
  1. The registered nurse taking shift report learns that an assigned patient is blind. How should the nurse best communicate with this patient?

A) Provide instructions in simple, clear terms.
B) Introduce herself in a firm, loud voice at the doorway of the room.
C) Lightly touch the patients arm and then introduce herself.
D) State her name and role immediately after entering the patients room.

A

D) State her name and role immediately after entering the patients room.

Feedback:
There are several guidelines to consider when interacting with a person who is blind or has low vision. Identify yourself by stating your name and role, before touching or making physical contact with the patient. When talking to the person, speak directly at him or her using a normal tone of voice. There is no need to raise your voice unless the person asks you to do so and there is no particular need to simplify verbal instructions.

How well did you know this?
1
Not at all
2
3
4
5
Perfectly
26
Q
  1. A female patient has been experiencing recurrent urinary tract infections. What health education should the nurse provide to this patient?

A) Bathe daily and keep the perineal region clean.
B) Avoid voiding immediately after sexual intercourse.
C) Drink liberal amounts of fluids.
D) Void at least every 6 to 8 hours.

A

C) Drink liberal amounts of fluids.

How well did you know this?
1
Not at all
2
3
4
5
Perfectly
27
Q
  1. A patient comes to the clinic complaining of pain in the epigastric region. What assessment question during the health interview would most help the nurse determine if the patient has a peptic ulcer?

A) Does your pain resolve when you have something to eat?
B) Do over-the-counter pain medications help your pain?
C) Does your pain get worse if you get up and do some exercise?
D) Do you find that your pain is worse when you need to have a bowel movement?

A

A) Does your pain resolve when you have something to eat?

Feedback:
Pain relief after eating is associated with duodenal ulcers. The pain of peptic ulcers is generally unrelated to activity or bowel function and may or may not respond to analgesics.

How well did you know this?
1
Not at all
2
3
4
5
Perfectly
28
Q
  1. A patient has presented at the clinic with symptoms of benign prostatic hyperplasia. What diagnostic findings would suggest that this patient has chronic urinary retention?

A) Hypertension
B) Peripheral edema
C) Tachycardia and other dysrhythmias
D) Increased blood urea nitrogen (BUN)

A

D) Increased blood urea nitrogen (BUN)

How well did you know this?
1
Not at all
2
3
4
5
Perfectly
29
Q
  1. You are providing care for an 82-year-old man whose signs and symptoms of Parkinson disease have become more severe over the past several months. The man tells you that he can no longer do as many things for himself as he used to be able to do. What factor should you recognize as impacting your patients life most significantly?

A) Neurologic deficits
B) Loss of independence
C) Age-related changes
D) Tremors and decreased mobility

A

B) Loss of independence

How well did you know this?
1
Not at all
2
3
4
5
Perfectly
30
Q
  1. A patient with ESKD receives continuous ambulatory peritoneal dialysis. The nurse observes that the dialysate drainage fluid is cloudy. What is the nurses most appropriate action?

A) Inform the physician and assess the patient for signs of infection.
B) Flush the peritoneal catheter with normal saline.
C) Remove the catheter promptly and have the catheter tip cultured.
D) Administer a bolus of IV normal saline as ordered.

A

A) Inform the physician and assess the patient for signs of infection.

How well did you know this?
1
Not at all
2
3
4
5
Perfectly
31
Q
  1. A clinic nurse is caring for a patient with a history of osteoporosis. Which of the following diagnostic tests best allows the care team to assess the patients risk of fracture?

A) Arthrography
B) Bone scan
C) Bone densitometry
D) Arthroscopy

A

C) Bone densitometry

Feedback:
Bone densitometry is used to detect bone density and can be used to assess the risk of fracture in osteoporosis. Arthrography is used to detect acute or chronic tears of joint capsule or supporting ligaments. Bone scans can be used to detect metastatic and primary bone tumors, osteomyelitis, certain fractures, and aseptic necrosis. Arthroscopy is used to visualize a joint.

How well did you know this?
1
Not at all
2
3
4
5
Perfectly
32
Q
  1. A 45-year-old woman comes into the health clinic for her annual check-up. She mentions to the nurse that she has noticed dimpling of the right breast that has occurred in a few months. What assessment would be most appropriate for the nurse to make?

A) Evaluate the patients milk production.
B) Palpate the area for a breast mass.
C) Assess the patients knowledge of breast cancer.
D) Assure the patient that this likely an age-related change.

A

B) Palpate the area for a breast mass.

How well did you know this?
1
Not at all
2
3
4
5
Perfectly
33
Q
  1. A nurse caring for a patient with diabetes insipidus is reviewing laboratory results. What is an expected urinalysis finding?

A) Glucose in the urine
B) Albumin in the urine
C) Highly dilute urine
D) Leukocytes in the urine

A

C) Highly dilute urine

Feedback: Patients with diabetes insipidus produce an enormous daily output of very dilute, water-like urine with a specific gravity of 1.001 to 1.005. The urine contains no abnormal substances such as glucose or albumin. Leukocytes in the urine are not related to the condition of diabetes insipidus, but would indicate a urinary tract infection, if present in the urine.

How well did you know this?
1
Not at all
2
3
4
5
Perfectly
34
Q
  1. The rehabilitation nurse is working closely with a patient who has a new orthosis following a knee injury. What are the nurses responsibilities to this patient? Select all that apply.

A) Help the patient learn to apply and remove the orthosis.
B) Teach the patient how to care for the skin that comes in contact with the orthosis.
C) Assist in the initial fitting of the orthosis.
D) Assist the patient in learning how to move the affected body part correctly.
E) Collaborate with the physical therapist to set goals for care.

A

A) Help the patient learn to apply and remove the orthosis.
B) Teach the patient how to care for the skin that comes in contact with the orthosis.
D) Assist the patient in learning how to move the affected body part correctly.
E) Collaborate with the physical therapist to set goals for care.

How well did you know this?
1
Not at all
2
3
4
5
Perfectly
35
Q
  1. The nurse is providing a health education workshop to a group of adults focusing on cancer prevention. The nurse should emphasize what action in order to reduce participants risks of renal carcinoma?

A) Avoiding heavy alcohol use
B) Control of sodium intake
C) Smoking cessation
D) Adherence to recommended immunization schedules

A

C) Smoking cessation

How well did you know this?
1
Not at all
2
3
4
5
Perfectly
36
Q
  1. Which of the following nurses actions carries the greatest potential to prevent hearing loss due to ototoxicity?

A) Ensure that patients understand the differences between sensory hearing loss and conductive
hearing loss.
B) Educate patients about expected age-related changes in hearing perception.
C) Educate patients about the risks associated with prolonged exposure to environmental noise.
D) Be aware of patients medication regimens and collaborate with other professionals accordingly.

A

D) Be aware of patients medication regimens and collaborate with other professionals accordingly.

How well did you know this?
1
Not at all
2
3
4
5
Perfectly
37
Q
  1. A 21-year-old woman has sought care because of heavy periods and has subsequently been diagnosed with menorrhagia. The nurse should recognize which of the following as the most likely cause of the patients health problem?

A) Hormonal disturbances
B) Cervical or uterine cancer
C) Pelvic inflammatory disease
D) A sexually transmitted infection (STI)

A

A) Hormonal disturbances

How well did you know this?
1
Not at all
2
3
4
5
Perfectly
38
Q
  1. A patient has suffered a muscle strain and is complaining of pain that she rates at 6 on a 10-point scale. The nurse should recommend what action?

A) Taking an opioid analgesic as ordered
B) Applying a cold pack to the injured site
C) Performing passive ROM exercises
D) Applying a heating pad to the affected muscle

A

B) Applying a cold pack to the injured site

How well did you know this?
1
Not at all
2
3
4
5
Perfectly
39
Q
  1. A patient is ready to be discharged home after a cataract extraction with intraocular lens implant and the nurse is reviewing signs and symptoms that need to be reported to the ophthalmologist immediately. Which of the patients statements best demonstrates an adequate understanding?

A) I need to call the doctor if I get nauseated.
B) I need to call the doctor if I have a light morning discharge.
C) I need to call the doctor if I get a scratchy feeling.
D) I need to call the doctor if I see flashing lights.

A

D) I need to call the doctor if I see flashing lights.

Feedback:
Postoperatively, the patient who has undergone cataract extraction with intraocular lens implant should report new floaters in vision, flashing lights, decrease in vision, pain, or increase in redness to the ophthalmologist. Slight morning discharge and a scratchy feeling can be expected for a few days. Blurring of vision may be experienced for several days to weeks.

How well did you know this?
1
Not at all
2
3
4
5
Perfectly
40
Q
  1. A patient has been admitted to the postsurgical unit following the creation of an ileal conduit. What should the nurse measure to determine the size of the appliance needed?

A) The circumference of the stoma
B) The narrowest part of the stoma
C) The widest part of the stoma
D) Half the width of the stoma

A

C) The widest part of the stoma

How well did you know this?
1
Not at all
2
3
4
5
Perfectly
41
Q
  1. The nurse is teaching breast self-examination (BSE) to a group of women. The nurse should recommend that the women perform BSE at what time?

A) At the time of menses
B) At any convenient time, regardless of cycles
C) Weekly
D) Between days 5 and 7 after menses

A

D) Between days 5 and 7 after menses

How well did you know this?
1
Not at all
2
3
4
5
Perfectly
42
Q
  1. Results of a patient barium swallow suggest that the patient has GERD. The nurse is planning health education to address the patients knowledge of this new diagnosis. Which of the following should the nurse encourage?

A) Eating several small meals daily rather than 3 larger meals
B) Keeping the head of the bed slightly elevated
C) Drinking carbonated mineral water rather than soft drinks
D) Avoiding food or fluid intake after 6:00 p.m.

A

B) Keeping the head of the bed slightly elevated

How well did you know this?
1
Not at all
2
3
4
5
Perfectly
43
Q
  1. A patient with ovarian cancer is admitted to the hospital for surgery and the nurse is completing the patients health history. What clinical manifestation would the nurse expect to assess?

A) Fish-like vaginal odor
B) Increased abdominal girth
C) Fever and chills
D) Lower abdominal pelvic pain

A

B) Increased abdominal girth

How well did you know this?
1
Not at all
2
3
4
5
Perfectly
44
Q
  1. A patient seeking care because of recurrent heartburn and regurgitation is subsequently diagnosed with a hiatal hernia. Which of the following should the nurse include in health education?

A) Drinking beverages after your meal, rather than with your meal, may bring some relief.
B) Its best to avoid dry foods, such as rice and chicken, because theyre harder to swallow.
C) Many patients obtain relief by taking over-the-counter antacids 30 minutes before eating.
D) Instead of eating three meals a day, try eating smaller amounts more often.

A

D) Instead of eating three meals a day, try eating smaller amounts more often.

Feedback:
Management for a hiatal hernia includes frequent, small feedings that can pass easily through the esophagus. Avoiding beverages and particular foods or taking OTC antacids are not noted to be beneficial.

How well did you know this?
1
Not at all
2
3
4
5
Perfectly
45
Q
  1. A nurse educator is teaching a group of recent nursing graduates about their occupational risks for contracting hepatitis B. What preventative measures should the educator promote? Select all that apply.
A)	Immunization
B)	Use of standard precautions
C)	Consumption of a vitamin-rich diet
D)	Annual vitamin K injections
E)	Annual vitamin B12 injections
A

A) Immunization

B) Use of standard precautions

How well did you know this?
1
Not at all
2
3
4
5
Perfectly
46
Q
  1. The clinic nurse is preparing a plan of care for a patient with a history of stress incontinence. What role will the nurse have in implementing a behavioral therapy approach?

A) Provide medication teaching related to pseudoephedrine sulfate.
B) Teach the patient to perform pelvic floor muscle exercises.
C) Prepare the patient for an anterior vaginal repair procedure.
D) Provide information on periurethral bulking.

A

B) Teach the patient to perform pelvic floor muscle exercises.

How well did you know this?
1
Not at all
2
3
4
5
Perfectly
47
Q
  1. An 80-year-old man in a long-term care facility has a chronic leg ulcer and states that the area has become increasingly painful in recent days. The nurse notes that the site is now swollen and warm to the touch. The patient should undergo diagnostic testing for what health problem?

A) Osteomyelitis
B) Osteoporosis
C) Osteomalacia
D) Septic arthritis

A

A) Osteomyelitis

Feedback: Infection is a risk after any surgery, but it is of particular concern for the postoperative orthopedic patient because of the risk of osteomyelitis. Orthopedic patients do not have an exaggerated risk of cellulitis, sepsis, or septic arthritis when compared to other surgical patients.

How well did you know this?
1
Not at all
2
3
4
5
Perfectly
48
Q
  1. A nurse is assessing a patient who has diabetes for the presence of peripheral neuropathy. The nurse should question the patient about what sign or symptom that would suggest the possible development of peripheral neuropathy?

A) Persistently cold feet
B) Pain that does not respond to analgesia
C) Acute pain, unrelieved by rest
D) The presence of a tingling sensation

A

D) The presence of a tingling sensation

How well did you know this?
1
Not at all
2
3
4
5
Perfectly
49
Q
  1. Following a motorcycle accident, a 17-year-old man is brought to the ED. What physical assessment findings related to the ear should be reported by the nurse immediately?

A) The malleus can be visualized during otoscopic examination.
B) The tympanic membrane is pearly gray.
C) Tenderness is reported by the patient when the mastoid area is palpated.
D) Clear, watery fluid is draining from the patients ear.

A

D) Clear, watery fluid is draining from the patients ear.

How well did you know this?
1
Not at all
2
3
4
5
Perfectly
50
Q
  1. Which of the following nursing interventions would most likely facilitate effective communication with a hearing-impaired patient?

A) Ask the patient to repeat what was said in order to evaluate understanding.
B) Stand directly in front of the patient to facilitate lip reading.
C) Reduce environmental noise and distractions before communicating.
D) Raise the voice to project sound at a higher frequency.

A

C) Reduce environmental noise and distractions before communicating.

How well did you know this?
1
Not at all
2
3
4
5
Perfectly
51
Q
  1. A nurse is caring for a patient with liver failure and is performing an assessment in the knowledge of the patients increased risk of bleeding. The nurse recognizes that this risk is related to the patients inability to synthesize prothrombin in the liver. What factor most likely contributes to this loss of function?

A) Alterations in glucose metabolism
B) Retention of bile salts
C) Inadequate production of albumin by hepatocytes
D) Inability of the liver to use vitamin K

A

D) Inability of the liver to use vitamin K

How well did you know this?
1
Not at all
2
3
4
5
Perfectly
52
Q
52.	A medical nurse is caring for a patient with type 1 diabetes. The patients medication administration record includes the administration of regular insulin three times daily. Knowing that the patients lunch tray will arrive at 11:45, when should the nurse administer the patients insulin?
A)	10:45
B)	11:15
C)	11:45
D)     11:50
A

B) 11:15

How well did you know this?
1
Not at all
2
3
4
5
Perfectly
53
Q
  1. A patient admitted with nephrotic syndrome is being cared for on the medical unit. When writing this patients care plan, based on the major clinical manifestation of nephrotic syndrome, what nursing diagnosis should the nurse include?

A) Constipation related to immobility
B) Risk for injury related to altered thought processes
C) Hyperthermia related to the inflammatory process
D) Excess fluid volume related to generalized edema

A

D) Excess fluid volume related to generalized edema

How well did you know this?
1
Not at all
2
3
4
5
Perfectly
54
Q
  1. A nurse is assessing a patients stoma on postoperative day 3. The nurse notes that the stoma has a shiny appearance and a bright red color. How should the nurse best respond to this assessment finding?

A) Irrigate the ostomy to clear a possible obstruction.
B) Contact the primary care provider to report this finding.
C) Document that the stoma appears healthy and well perfused.
D) Document a nursing diagnosis of Impaired Skin Integrity.

A

C) Document that the stoma appears healthy and well perfused.

How well did you know this?
1
Not at all
2
3
4
5
Perfectly
55
Q
  1. The health care team is caring for a patient with osteomalacia. It has been determined that the osteomalacia is caused by malabsorption. What is the usual treatment for osteomalacia caused by malabsorption?

A) Supplemental calcium and increased doses of vitamin
B) Exogenous parathyroid hormone and multivitamins
C) Colony-stimulating factors and calcitonin
D) Supplemental potassium and pancreatic enzymes

A

A) Supplemental calcium and increased doses of vitamin D

How well did you know this?
1
Not at all
2
3
4
5
Perfectly
56
Q
  1. A patient has had a cast placed for the treatment of a humeral fracture. The nurses most recent assessment shows signs and symptoms of compartment syndrome. What is the nurses most appropriate action?

A) Arrange for a STAT assessment of the patients serum calcium levels.
B) Perform active range of motion exercises.
C) Assess the patients joint function symmetrically.
D) Contact the primary care provider immediately.

A

D) Contact the primary care provider immediately.

Feedback:
This major neurovascular problem is caused by pressure within a muscle compartment that increases to such an extent that microcirculation diminishes, leading to nerve and muscle anoxia and necrosis. Function can be permanently lost if the anoxic situation continues for longer than 6 hours. Therefore, immediate medical care is a priority over further nursing assessment. Assessment of calcium levels is unnecessary.

How well did you know this?
1
Not at all
2
3
4
5
Perfectly
57
Q
  1. A patient with portal hypertension has been admitted to the medical floor. The nurse should prioritize which of the following assessments related to the manifestations of this health problem?

A) Assessment of blood pressure and assessment for headaches and visual changes
B) Assessments for signs and symptoms of venous thromboembolism
C) Daily weights and abdominal girth measurement
D) Blood glucose monitoring q4h

A

C) Daily weights and abdominal girth measurement

How well did you know this?
1
Not at all
2
3
4
5
Perfectly
58
Q
  1. A patient diagnosed with arthritis has been taking aspirin and now reports experiencing tinnitus and hearing loss. What should the nurse teach this patient?

A) The hearing loss will likely resolve with time after the drug is discontinued.
B) The patients hearing loss and tinnitus are irreversible at this point.
C) The patients tinnitus is likely multifactorial, and not directly related to aspirin use.
D) The patients tinnitus will abate as tolerance to aspirin develops.

A

A) The hearing loss will likely resolve with time after the drug is discontinued.

How well did you know this?
1
Not at all
2
3
4
5
Perfectly
59
Q
  1. A patient who had surgery for gallbladder disease has just returned to the postsurgical unit from postanesthetic recovery. The nurse caring for this patient knows to immediately report what assessment finding to the physician?

A) Decreased breath sounds
B) Drainage of bile-colored fluid onto the abdominal dressing
C) Rigidity of the abdomen
D) Acute pain with movement

A

C) Rigidity of the abdomen

How well did you know this?
1
Not at all
2
3
4
5
Perfectly
60
Q
  1. A patient tells the nurse that he has pain and numbness to his thumb, first finger, and second finger of the right hand. The nurse discovers that the patient is employed as an auto mechanic, and that the pain is increased while working. This may indicate that the patient could possibly have what health problem?

A) Carpel tunnel syndrome
B) Tendonitis
C) Impingement syndrome
D) Dupuytrens contracture

A

A) Carpel tunnel syndrome

How well did you know this?
1
Not at all
2
3
4
5
Perfectly
61
Q
  1. A patient with end-stage liver disease has developed hypervolemia. What nursing interventions would be most appropriate when addressing the patients fluid volume excess? Select all that apply.

A) Administering diuretics
B) Administering calcium channel blockers
C) Implementing fluid restrictions
D) Implementing a 1500 kcal/day restriction
E) Enhancing patient positioning

A

A) Administering diuretics
C) Implementing fluid restrictions
E) Enhancing patient positioning

How well did you know this?
1
Not at all
2
3
4
5
Perfectly
62
Q
  1. A patient with a diagnosis of retinal detachment has undergone a vitreoretinal procedure on an outpatient basis. What subject should the nurse prioritize during discharge education?

A) Risk factors for postoperative cytomegalovirus (CMV)
B) Compensating for vision loss for the next several weeks
C) Non-pharmacologic pain management strategies
D) Signs and symptoms of increased intraocular pressure

A

D) Signs and symptoms of increased intraocular pressure

How well did you know this?
1
Not at all
2
3
4
5
Perfectly
63
Q
  1. A 17-year-old girl has come to the free clinic for her annual examination. She tells the nurse she uses tampons and asks how long she may safely leave her tampon in place. What is the nurses best response?

A) You may leave the tampon in overnight.
B) The tampon should be changed at least twice per day.
C) Tampons are dangerous and, ideally, you should not be using them.
D) Tampons need to be changed every 4 to 6 hours.

A

D) Tampons need to be changed every 4 to 6 hours.

How well did you know this?
1
Not at all
2
3
4
5
Perfectly
64
Q
  1. A 16-year-old presents at the emergency department complaining of right lower quadrant pain and is subsequently diagnosed with appendicitis. When planning this patients nursing care, the nurse should prioritize what nursing diagnosis?

A) Imbalanced Nutrition: Less Than Body Requirements Related to Decreased Oral Intake
B) Risk for Infection Related to Possible Rupture of Appendix
C) Constipation Related to Decreased Bowel Motility and Decreased Fluid Intake
D) Chronic Pain Related to Appendicitis

A

B) Risk for Infection Related to Possible Rupture of Appendix

How well did you know this?
1
Not at all
2
3
4
5
Perfectly
65
Q
  1. A patient has been brought to the emergency department with abdominal pain and is subsequently diagnosed with appendicitis. The patient is scheduled for an appendectomy but questions the nurse about how his health will be affected by the absence of an appendix. How should the nurse best respond?

A) Your appendix doesnt play a major role, so you wont notice any difference after you recovery from surgery.
B) The surgeon will encourage you to limit your fat intake for a few weeks after the surgery, but your body will then begin to compensate.
C) Your body will absorb slightly fewer nutrients from the food you eat, but you wont be aware of this.
D) Your large intestine will adapt over time to the absence of your appendix.

A

A) Your appendix doesnt play a major role, so you wont notice any difference after you recovery from surgery.

How well did you know this?
1
Not at all
2
3
4
5
Perfectly
66
Q
  1. A hospitalized patient with impaired vision must get a picture in his or her mind of the hospital room and its contents in order to mobilize independently and safely. What must the nurse monitor in the patients room?

A) That a commode is always available at the bedside
B) That all furniture remains in the same position
C) That visitors do not leave items on the bedside table
D) That the patients slippers stay under the bed

A

B) That all furniture remains in the same position

How well did you know this?
1
Not at all
2
3
4
5
Perfectly
67
Q
  1. Several residents of a long-term care facility have developed signs and symptoms of viral conjunctivitis. What is the most appropriate action of the nurse who oversees care in the facility?

A) Arrange for the administration of prophylactic antibiotics to unaffected residents.
B) Instill normal saline into the eyes of affected residents two to three times daily.
C) Swab the conjunctiva of unaffected residents for culture and sensitivity testing.
D) Isolate affected residents from residents who have not developed conjunctivitis.

A

D) Isolate affected residents from residents who have not developed conjunctivitis.

Feedback:
To prevent spread during outbreaks of conjunctivitis caused by adenovirus, health care facilities must set aside specified areas for treating patients diagnosed with or suspected of having conjunctivitis caused by adenovirus. Antibiotics and saline flushes are ineffective and normally no need to perform testing of individuals lacking symptoms.

How well did you know this?
1
Not at all
2
3
4
5
Perfectly
68
Q
  1. The admissions department at a local hospital is registering an elderly man for an outpatient diagnostic test. The admissions nurse asks the man if he has an advanced directive. The man responds that he does not want to complete an advance directive because he does not want anyone controlling his finances. What would be appropriate information for the nurse to share with this patient?

A) Advance directives are not legal documents, so you have nothing to worry about.
B) Advance directives are limited only to health care instructions and directives.
C) Your finances cannot be managed without an advance directive.
D) Advance directives are implemented when you become incapacitated, and then you will use a living will to allow the state to manage your money.

A

B) Advance directives are limited only to health care instructions and directives.

How well did you know this?
1
Not at all
2
3
4
5
Perfectly
69
Q
  1. A gerontologic nurse is assessing a patient who has numerous comorbid health problems. What assessment findings should prompt the nurse to suspect a UTI? Select all that apply.
A)	Food cravings
B)	Upper abdominal pain
C)	Insatiable thirst
D)	Uncharacteristic fatigue
E)	New onset of confusion
A

D) Uncharacteristic fatigue

How well did you know this?
1
Not at all
2
3
4
5
Perfectly
70
Q
  1. The nurse is assessing a patient suspected of having developed acute glomerulonephritis. The nurse should expect to address what clinical manifestation that is characteristic of this health problem?

A) Hematuria
B) Precipitous decrease in serum creatinine levels
C) Hypotension unresolved by fluid administration
D) Glucosuria

A

A) Hematuria

How well did you know this?
1
Not at all
2
3
4
5
Perfectly
71
Q
  1. a patient has been admitted to the medical unit for the treatment of osteoporosis. when reviewing the medication administration record, the nurse should anticipate a bisphosphonate medication to be ordered.
A

Bisphosphonates such as Fosamax increase bone mass and decrease bone loss by inhibiting osteoclast function

How well did you know this?
1
Not at all
2
3
4
5
Perfectly
72
Q
  1. The school nurse is presenting a class on female reproductive health. The nurse should describe what aspect of Pap smears?

A) The test may be performed at any time during the patients menstrual cycle.
B) The smear should be done every 2 years.
C) The test can detect early evidence of cervical cancer.
D) Falsepositive Pap smear results occur mostly from not douching before the examination.

A

C) The test can detect early evidence of cervical cancer.

How well did you know this?
1
Not at all
2
3
4
5
Perfectly
73
Q
  1. A nurse who provides care in a walk-in clinic assesses a wide range of individuals. The nurse should identify which of the following patients as having the highest risk for chronic pancreatitis?

A) A 45-year-old obese woman with a high-fat diet
B) An 18-year-old man who is a weekend binge drinker
C) A 39-year-old man with chronic alcoholism
D) A 51-year-old woman who smokes one-and-a-half packs of cigarettes per day

A

C) A 39-year-old man with chronic alcoholism

How well did you know this?
1
Not at all
2
3
4
5
Perfectly
74
Q
  1. A patient has had a brace prescribed to facilitate recovery from a knee injury. What are the potential therapeutic benefits of a brace? Select all that apply.
A)	Preventing additional injury
B)	Immobilizing prior to surgery
C)	Providing support
D)	Controlling movement
E)	Promoting bone remodeling
A

A) Preventing additional injury
C) Providing support
D) Controlling movement

How well did you know this?
1
Not at all
2
3
4
5
Perfectly
75
Q
  1. The nurse is caring for a patient who has a fluid volume deficit. When evaluating this patients urinalysis results, what should the nurse anticipate?

A) A fluctuating urine specific gravity
B) A fixed urine specific gravity
C) A decreased urine specific gravity
D) An increased urine specific gravity

A

D) An increased urine specific gravity

How well did you know this?
1
Not at all
2
3
4
5
Perfectly
76
Q
  1. A nurse is creating a care plan for a patient who is receiving parenteral nutrition. The patients care plan should include nursing actions relevant to what potential complications? Select all that apply.
A)	Dumping syndrome
B)	Clotted or displaced catheter
C)	Pneumothorax
D)	Hyperglycemia
E)	Line sepsis
A

B) Clotted or displaced catheter
C) Pneumothorax
D) Hyperglycemia
E) Line sepsis

How well did you know this?
1
Not at all
2
3
4
5
Perfectly
77
Q
  1. A nurse is assessing a patient who is receiving traction. The nurses assessment confirms that the patient is able to perform plantar flexion. What conclusion can the nurse draw from this finding?

A) The leg that was assessed is free from DVT.
B) The patients tibial nerve is functional.
C) Circulation to the distal extremity is adequate.
D) The patient does not have peripheral neurovascular dysfunction.

A

B) The patients tibial nerve is functional.

How well did you know this?
1
Not at all
2
3
4
5
Perfectly
78
Q
  1. A nurse is planning the care of a patient who will require a prolonged course of skeletal traction. When planning this patients care, the nurse should prioritize interventions related to which of the following risk nursing diagnoses?

A) Risk for Impaired Skin Integrity
B) Risk for Falls
C) Risk for Imbalanced Fluid Volume
D) Risk for Aspiration

A

A) Risk for Impaired Skin Integrity

How well did you know this?
1
Not at all
2
3
4
5
Perfectly
79
Q
  1. A patient with a diagnosis of syndrome of inappropriate antidiuretic hormone secretion (SIADH) is being cared for on the critical care unit. The priority nursing diagnosis for a patient with this condition is what?

A) Risk for peripheral neurovascular dysfunction
B) Excess fluid volume
C) Hypothermia
D) Ineffective airway clearance

A

B) Excess fluid volume

How well did you know this?
1
Not at all
2
3
4
5
Perfectly
80
Q
  1. An older adult patient experienced a fall and required treatment for a fractured hip on the orthopedic unit. Which of the following are contributory factors to the incidence of falls and fractured hips among the older adult population? Select all that apply.
A)	Loss of visual acuity
B)	Adverse medication effects
C)	Slowed reflexes
D)	Hearing loss
E)	Muscle weakness
A

A) Loss of visual acuity
B) Adverse medication effects
C) Slowed reflexes
E) Muscle weakness

How well did you know this?
1
Not at all
2
3
4
5
Perfectly
81
Q
  1. A patient has been admitted to the post-surgical unit following a thyroidectomy. To promote comfort and safety, how should the nurse best position the patient?

A) Side-lying (lateral) with one pillow under the head
B) Head of the bed elevated 30 degrees and no pillows placed under the head
C) Semi-Fowlers with the head supported on two pillows
D) Supine, with a small roll supporting

A

C) Semi-Fowlers with the head supported on two pillows

How well did you know this?
1
Not at all
2
3
4
5
Perfectly
82
Q
  1. A woman scheduled for a simple mastectomy in one week is having her preoperative education provided by the clinic nurse. What educational intervention will be of primary importance to prevent hemorrhage in the postoperative period?

A) Limit her intake of green leafy vegetables.
B) Increase her water intake to 8 glasses per day.
C) Stop taking aspirin.
D) Have nothing by mouth for 6 hours before surgery.

A

C) Stop taking aspirin.

How well did you know this?
1
Not at all
2
3
4
5
Perfectly
83
Q
  1. A nurse is caring for an older adult who has been diagnosed with geriatric failure to thrive. This patients prolonged immobility creates a risk for what complication?

A) Muscle clonus
B) Muscle atrophy
C) Rheumatoid arthritis
D) Muscle fasciculations

A

B) Muscle atrophy

How well did you know this?
1
Not at all
2
3
4
5
Perfectly
84
Q
  1. A patients health history is suggestive of inflammatory bowel disease. Which of the following would suggest Crohns disease, rather that ulcerative colitis, as the cause of the patients signs and symptoms?

A) A pattern of distinct exacerbations and remissions
B) Severe diarrhea
C) An absence of blood in stool
D) Involvement of the rectal mucosa

A

C) An absence of blood in stool

85
Q
  1. A nurse is preparing to place a patients ordered nasogastric tube. How should the nurse best determine the correct length of the nasogastric tube?

A) Place distal tip to nose, then ear tip and end of xiphoid process.
B) Instruct the patient to lie prone and measure tip of nose to umbilical area.
C) Insert the tube into the patients nose until secretions can be aspirated.
D) Obtain an order from the physician for the length of tube to insert.

A

A) Place distal tip to nose, then ear tip and end of xiphoid process.

86
Q
  1. A patient has been successfully treated for kidney stones and is preparing for discharge. The nurse recognizes the risk of recurrence and has planned the patients discharge education accordingly. What preventative measure should the nurse encourage the patient to adopt?

A) Increasing intake of protein from plant sources
B) Increasing fluid intake
C) Adopting a high-calcium diet
D) Eating several small meals

A

B) Increasing fluid intake

Feedback:
Increased fluid intake is encouraged to prevent the recurrence of kidney stones. Protein intake from all sources should be limited. Most patients do not require a low-calcium diet, but increased calcium intake would be contraindicated for all patients. Eating small, frequent meals does not influence the risk for recurrence.

87
Q
  1. A patient who presents for an eye examination is diagnosed as having a visual acuity of 20/40. The patient asks the nurse what these numbers specifically mean. What is a correct response by the nurse?

A) A person whose vision is 20/40 can see an object from 40 feet away that a person with 20/20 vision can see from 20 feet away.
B) A person whose vision is 20/40 can see an object from 20 feet away that a person with 20/20 vision can see from 40 feet away.
C) A person whose vision is 20/40 can see an object from 40 inches away that a person with 20/20 vision can see from 20 inches away.
D) A person whose vision is 20/40 can see an object from 20 inches away that a person with 20/20 vision can see from 40 inches away.

A

B) A person whose vision is 20/40 can see an object from 20 feet away that a person with 20/20 vision can see from 40 feet away.

88
Q
  1. The nurse is providing patient teaching to a patient with early stage Alzheimers disease (AD) and her family. The patient has been prescribed donepezil hydrochloride (Aricept). What should the nurse explain to the patient and family about this drug?

A) It slows the progression of AD.
B) It cures AD in a small minority of patients.
C) It removes the patients insight that he or she has AD.
D) It limits the physical effects of AD and other dementias.

A

A) It slows the progression of AD.

89
Q
  1. A patient is exploring treatment options after being diagnosed with age-related cataracts that affect her vision. What treatment is most likely to be used in this patients care?

A) Antioxidant supplements, vitamin C and E, beta-carotene, and selenium
B) Eyeglasses or magnifying lenses
C) Corticosteroid eye drops
D) Surgical intervention

A

D) Surgical intervention

90
Q
  1. A nurse is providing care for a patient who has a diagnosis of irritable bowel syndrome (IBS). When planning this patients care, the nurse should collaborate with the patient and prioritize what goal?

A) Patient will accurately identify foods that trigger symptoms.
B) Patient will demonstrate appropriate care of his ileostomy.
C) Patient will demonstrate appropriate use of standard infection control precautions.
D) Patient will adhere to recommended guidelines for mobility and activity.

A

A) Patient will accurately identify foods that trigger symptoms.

Feedback:
A major focus of nursing care for the patient with IBS is to identify factors that exacerbate symptoms. Surgery is not used to treat this health problem and infection control is not a concern that is specific to this diagnosis. Establishing causation likely is more important to the patient than managing physical activity.

91
Q
  1. A female patient tells the nurse that she thinks she has a vaginal infection because she has noted inflammation of her vulva and the presence of a frothy, yellow-green discharge. The nurse recognizes that the clinical manifestations described are typical of what vaginal infection?

A) Trichomonas vaginalis
B) Candidiasis
C) Gardnerella
D) Gonorrhea

A

A) Trichomonas vaginalis

92
Q
  1. A patient with genital herpes is having an acute exacerbation. What medication would the nurse expect to be ordered to suppress the symptoms and shorten the course of the infection?

A) Clotrimazole (Gyne-Lotrimin)
B) Metronidazole (Flagyl)
C) Podophyllin (Podofin)
D) Acyclovir (Zovirax)

A

D) Acyclovir (Zovirax)

93
Q
  1. The management of the patients gastrostomy is an assessment priority for the home care nurse. What statement would indicate that the patient is managing the tube correctly?

A) I clean my stoma twice a day with alcohol.
B) The only time I flush my tube is when Im putting in medications.
C) I flush my tube with water before and after each of my medications.
D) I try to stay still most of the time to avoid dislodging my tube.

A

C) I flush my tube with water before and after each of my medications.

94
Q
  1. Following a recent history of dyspareunia and lower abdominal pain, a patient has received a diagnosis of pelvic inflammatory disease (PID). When providing health education related to self-care, the nurse should address which of the following topics? Select all that apply.

A) Use of condoms to prevent infecting others
B) Appropriate use of antibiotics
C) Taking measures to prevent pregnancy
D) The need for a Pap smear every 3 months
E) The importance of weight loss in preventing symptoms

A

A) Use of condoms to prevent infecting others

B) Appropriate use of antibiotics

95
Q
  1. A patient who experienced an upper GI bleed due to gastritis has had the bleeding controlled and the patients condition is now stable. For the next several hours, the nurse caring for this patient should assess for what signs and symptoms of recurrence?

A) Tachycardia, hypotension, and tachypnea
B) Tarry, foul-smelling stools
C) Diaphoresis and sudden onset of abdominal pain
D) Sudden thirst, unrelieved by oral fluid administration

A

A) Tachycardia, hypotension, and tachypnea

96
Q
  1. A patient with renal failure secondary to diabetic nephropathy has been admitted to the medical unit. What is the most life-threatening effect of renal failure for which the nurse should monitor the patient?

A) Accumulation of wastes
B) Retention of potassium
C) Depletion of calcium
D) Lack of BP control

A

B) Retention of potassium

97
Q
  1. A nurse practitioner is assessing a 55-year-old male patient who is complaining of perineal discomfort, burning, urgency, and frequency with urination. The patient states that he has pain with ejaculation. The nurse knows that the patient is exhibiting symptoms of what?

A) Varicocele
B) Epididymitis
C) Prostatitis
D) Hydrocele

A

C) Prostatitis

98
Q
  1. A patient with thyroid cancer has undergone surgery and a significant amount of parathyroid tissue has been removed. The nurse caring for the patient should prioritize what question when addressing potential complications?

A) Do you feel any muscle twitches or spasms?
B) Do you feel flushed or sweaty?
C) Are you experiencing any dizziness or lightheadedness?
D) Are you having any pain that seems to be radiating from your bones?

A

A) Do you feel any muscle twitches or spasms?

99
Q
  1. A patient is receiving education about his upcoming Billroth I procedure (gastroduodenostomy). This patient should be informed that he may experience which of the following adverse effects associated with this procedure?

A) Persistent feelings of hunger and thirst
B) Constipation or bowel incontinence
C) Diarrhea and feelings of fullness
D) Gastric reflux and belching

A

C) Diarrhea and feelings of fullness

100
Q
  1. A patient with gastroesophageal reflux disease (GERD) has a diagnosis of Barretts esophagus with minor cell changes. Which of the following principles should be integrated into the patients subsequent care?

A) The patient will require an upper endoscopy every 6 months to detect malignant changes.
B) Liver enzymes must be checked regularly, as H2 receptor antagonists may cause hepatic damage.
C) Small amounts of blood are likely to be present in the stools and are not cause for concern.
D) Antacids may be discontinued when symptoms of heartburn subside.

A

A) The patient will require an upper endoscopy every 6 months to detect malignant changes.

101
Q

The nurse is preparing a teaching plan for a client who had a cataract extraction with intraocular implantation. Which home care measures should the nurse include in the plan? Select all that apply.

a) Avoid activities that require bending over.
b) Contact the surgeon if eye scratchiness occurs.
c) Take acetaminophen for minor eye discomfort.
d) Expect episodes of sudden severe pain in the eye.
e) Place an eye shield on the surgical eye at bedtime.
f) Contact the surgeon if a decrease in visual acuity occurs.

A

a) Avoid activities that require bending over.
b) Contact the surgeon if eye scratchiness occurs.
c) Take acetaminophen for minor eye discomfort.
e) Place an eye shield on the surgical eye at bedtime.
f) Contact the surgeon if a decrease in visual acuity occurs.

102
Q

The nurse is caring for a client with a pelvic fracture. Which is the nurse’s priority action to prevent complications?

a. Monitor temperature daily.
b. Insert a urethral catheter.
c. Monitor blood pressure frequently.
d. Turn the client every 2 hours.

A

c. Monitor blood pressure frequently.

Feedback: With a pelvic fracture, internal organ damage may result in bleeding and hypovolemic shock. The nurse monitors the client’s vital signs, skin color, and level of consciousness frequently to determine whether shock is occurring

103
Q

After surgery for removal of cataract, a client is being discharged, and the nurse has completed discharge instruction. Which client statement indicates that the outcome of the teaching plan has been met?

a. “Dots or flashing lights in my vision are to be expected for the first few days.”
b. “I should avoid pulling or pushing any object that weighs more than 15 lbs.”
c. “I need to keep the eye patch on for about a week after surgery.”
d. “I need to wear sunglasses for the first 3 to 4 days even when I’m inside.”

A

b. “I should avoid pulling or pushing any object that weighs more than 15 lbs.”

Explanation:
After cataract surgery, the client needs to avoid lifting, pulling, or pushing any object that weighs more than 15 pounds to prevent putting excessive pressure on the surgical site. Sunglasses should be worn when outdoors during the day because the eye is sensitive to light. Dots, flashing lights, a decrease in vision, pain, and increased redness need to be reported to the physician immediately. The eye patch is worn for 24 hours after surgery, followed by eyeglasses worn during the day and a metal shield worn at night for 1 to 4 weeks.

104
Q
A patient is being discharged home from the ambulatory surgical center after cataract surgery. In reviewing the discharge instructions with the patient, the nurse instructs the patient to immediately call the office if the patient experiences what?
A) Slight morning discharge from the eye
B) Any appearance of redness of the eye
C) A "scratchy" feeling in the eye
D) A new floater in vision
A

D) A new floater in vision

Feedback:
Cataract surgery increases the risk of retinal detachment and the patient must be instructed to notify the surgeon of new floaters in vision, flashing lights, decrease in vision, pain, or increase in redness. Slight morning discharge, some redness, and a scratchy feeling may be expected for a few days after surgery.

105
Q

Which statement indicates that the client understands teaching about the use of aspirin post–cataract surgery?

a. “It may increase intraocular pressure after cataract surgery.”
b. “It changes the ability of the blood to clot and increases the risk of bleeding.”
c. “It reduces inflammation and might mask any symptoms of infection.”
d. “It can cause nausea and vomiting and may increase intraocular pressure.”

A

b. “It changes the ability of the blood to clot and increases the risk of bleeding.”

106
Q

A client with chronic open-angle glaucoma is now presenting with eye pain and intraocular pressure of 50 mm Hg. An immediate iridotomy is scheduled. Which of the following describes the desired effects of this procedure?

a. Reverse optic nerve damage
b. Improve outflow drainage
c. To relieve pain
d. Restore vision

A

b. Improve outflow drainage

Explanation:
Laser iridotomy or standard iridotomy is a surgical procedure that provides additional outlet drainage of aqueous humor. This is done to lower the IOP as quickly as possible since permanent vision loss can occur in 1 to 2 days. Once optic nerve damage occurs, it cannot be reversed, and vision is not restored. Pain that occurs with rising IOP will be controlled once pressure is lowered through improved outflow drainage.

107
Q

The nurse assesses several clients. Which one is most likely to have secondary open-angle glaucoma?

a. Client with gradual onset of blurred vision
b. Client who has recently had eye surgery
c. Client who sees halos around lights
d. Client with reactive pupils and clear sclera

A

b. Client who has recently had eye surgery

108
Q

A client with acute-angle glaucoma has several medications ordered. Which medications does the nurse question? (Select all that apply.)

a. Acetazolamide (Diamox)
b. Pilocarpine (Pilocar)
c. Atropine (Isopto Atropine)
d. Latanoprost (Xalatan)
e. Timolol (Timoptic)
f. Epinephrine

A

c. Atropine (Isopto Atropine)

f. Epinephrine

109
Q
  1. A patient with chronic open-angle glaucoma is being taught to self-administer pilocarpine. After the patient administers the pilocarpine, the patient states that her vision is blurred. Which nursing action is most appropriate?
    A) Holding the next dose and notifying the physician
    B) Treating the patient for an allergic reaction
    C) Suggesting that the patient put on her glasses
    D) Explaining that this is an expected adverse effect
A

D) Explaining that this is an expected adverse effect

Feedback:
Pilocarpine, a miotic drug used to treat glaucoma, achieves its effect by constricting the pupil. Blurred vision lasting 1 to 2 hours after instilling the eye drops is an expected adverse effect. The patient may also note difficulty adapting to the dark. Because blurred vision is an expected adverse effect, the drug does not need to be withheld, nor does the physician need to be notified. Likewise, the patient does not need to be treated for an allergic reaction. Wearing glasses will not alter this temporary adverse effect.

110
Q

Which client statement would lead the nurse to suspect that the client is experiencing bacterial conjunctivitis?

a. “My eyes feel like they are on fire.”
b. “My eyelids were stuck together this morning.”
c. “It feels like there is something stuck in my eye.”
d. “My eyes hurt when I’m in the bright sunlight.”

A

b. “My eyelids were stuck together this morning.”

Explanation:
Burning, a sensation of a foreign body, and pain in bright light (photophobia) are signs and symptoms associated with any type of conjunctivitis. The drainage related to bacterial conjunctivitis is usually present in the morning, and the eyes may be difficult to open becacuse of adhesions caused by the exudate.

111
Q

The nurse is administering an ophthalmic ointment to a patient with conjunctivitis. What disadvantage of the application of an ointment does the nurse explain to the patient?

a. It does not work as rapidly as eye drops do.
b. Blurred vision results after application.
c. It has a lower concentration than eye drops.
d. It has more side effects than eye drops.

A

b. Blurred vision results after application.

Explanation:
Ophthalmic ointments have extended retention time in the conjunctival sac and provide a higher concentration than eye drops. The major disadvantage of ointments is the blurred vision that results after application. In general, eyelids and eyelid margins are best treated with ointments.

112
Q

A client comes to the eye clinic for an examination. The client tells the nurse that his vision is like a target with the bull’s eye area of the image missing. What would the nurse suspect?

a. Retinal detachment
b. Fractured orbit
c. Conjunctivitis
d. Macular degeneration

A

d. Macular degeneration

Explanation:
When the macula becomes irreparably damaged, clients compare their vision to a target in which the bull’s-eye area of the image is absent. Retinal detachment, a fractured orbit, and conjunctivitis do not present with vision likened to a target with the bull’s eye portion missing.

113
Q

A patient is to have an angiography done using fluorescein as a contrast agent to determine if the patient has macular edema. What laboratory work should the nurse monitor prior to the angiography?

a. Hemoglobin and hematocrit
b. BUN and creatinine
c. Platelet count
d. AST and ALT

A

b. BUN and creatinine

Explanation:
Angiography is done using fluorescein or indocyanine green as contrast agents. Fluorescein angiography is used to evaluate clinically significant macular edema, document macular capillary nonperfusion, and identify retinal and choroidal neovascularization (growth of abnormal new blood vessels) in age-related macular degeneration. It is an invasive procedure in which fluorescein dye is injected, usually into an antecubital vein. Prior to the angiography, the patient’s blood urea nitrogen (BUN) and creatinine should be checked to ensure that the kidneys will excrete the contrast agent

114
Q

A client has been referred to an ophthalmologist for suspected macular degeneration. The nurse knows to prepare what test for the physician to give the client?

a. Visual field
b. Amsler grid
c. Slit lamp
d. Ishihara polychromatic plates

A

b. Amsler grid

Explanation:
Clients with macular problems are tested with an Amsler grid. It is made up of a geometric grid of identical squares with a central fixation point. The examiner instructs the client to stare at the central fixation spot on the grid and report if they see any distortion of the squares. Clients with macular problems may say some of the squares are faded or wavy.

An Ishihara polychromatic plate, visual field, or slit lamp test will not diagnose macular degeneration.

115
Q

The nures is assessing a client using an Amsler Grid. The nurse is assessing for which of the following?

a. Visual field
b. Macular problems
c. Visual acuity
d. Intraocular pressure

A

b. Macular problems

Explanation:
The Amsler grid is a test used to assess clients for macular problems. Visual acuity is tested using the Snellen chart. Intraocular pressure is measured using tonometry. Perimetry testing evaluates the field of vision.

116
Q

The nurse is doing discharge teaching with a client newly diagnosed with Ménière’s disease. Why would the nurse advise a low-sodium diet to this client?

A

To reduce the production of fluid in the inner ear

Feedback:
A low-sodium diet lessens edema. This measure does not help minimize the adverse effects of drug therapy, reduce the magnitude of the hearing deficit, or minimize the risk of a tumor that involves the vestibulocochlear nerve.

117
Q

A dietary modification for a patient with Ménière’s disease would be:

A

A decrease in sodium intake to 2,000 mg daily.

Patients with Ménière’s disease can be successfully treated by adhering to a low-sodium (2000 mg/day) diet, with no caffeine and alcohol.

118
Q

The nurse is caring for a patient with Ménière’s disease who is hospitalized with severe vertigo. What medication does the nurse anticipate administering to shorten the attack?

A

Meclizine (Antivert)

Pharmacologic therapy for Ménière’s disease consists of antihistamines, such as meclizine, which shortens the attack (NIDCD, 2010).

119
Q
  1. An 11-year-old boy has been brought to the ED by his teacher, who reports that the boy may be having a really bad allergic reaction to peanuts after trading lunches with a peer. The triage nurses rapid assessment reveals the presence of respiratory and cardiac arrest. What interventions should the nurse prioritize?
    A) Establishing central venous access and beginning fluid resuscitation
    B) Establishing a patent airway and beginning cardiopulmonary resuscitation
    C) Establishing peripheral IV access and administering IV epinephrine
    D) Performing a comprehensive assessment and initiating rapid fluid replacement
A

B) Establishing a patent airway and beginning cardiopulmonary resuscitation

FEEDBACK:
If cardiac arrest and respiratory arrest are imminent or have occurred, CPR is performed. As well, a patent airway is an immediate priority. Epinephrine is not withheld pending IV access and fluid resuscitation is not a priority.

120
Q

A triage nurse in the ED is on shift when a grandfather carries his 4-year-old grandson into the ED. The child is not breathing, and the grandfather states the boy was stung by a bee in a nearby park while they were waiting for the boys mother to get off work. Which of the following would lead the nurse to suspect that the boy is experiencing anaphylactic shock?
A. Rapid onset of acute hypertension
B. Rapid onset of respiratory distress
C. Rapid onset of neurologic compensation
D. Rapid onset of cardiac arrest

A

B. Rapid onset of respiratory distress

Feedback:
Characteristics of severe anaphylaxis usually include rapid onset of hypotension, neurologic compromise, and respiratory distress. Cardiac arrest can occur if prompt treatment is not provided.

121
Q
The nurse admits a patient to the PACU with a blood pressure of 132/90 mm Hg and a pulse of 68 beats per minute. After 30 minutes, the patients blood pressure is 94/47 mm Hg, and the pulse is 110. The nurse documents that the patients skin is cold, moist, and pale. Of what is the patient showing signs?
A.	Hypothermia 
B.	Hypovolemic shock 
C.	Neurogenic shock Malignant
D.	 hyperthermia
A

B. Hypovolemic shock

FEEDBACK:
The patient is exhibiting symptoms of hypovolemic shock; therefore, the nurse should notify the patients physician and anticipate orders for fluid and/or blood product replacement. Neurogenic shock does not normally result in tachycardia and malignant hyperthermia would not present at this stage in the operative experience. Hypothermia does not cause hypotension and tachycardia.

122
Q

3.As the nurse working in a gerontology clinic, you know that some elderly people do not adhere to therapeutic regimens because of chronic illnesses that require long-term treatment by several health care providers. What is the most important consideration when dealing with this segment of the population?
A) Health care professionals must know all the dietary supplements the patient is taking.
B) Health care professionals must work together to provide coordinated care.
C) Health care professionals may negate the efforts of another health care provider.
D) Health care professionals must have a peer witness their interactions with the patient.

A

B) Health care professionals must work together to provide coordinated care.

123
Q
  1. A patient tells the nurse that her doctor just told her that her new diagnosis of rheumatoid arthritis is considered to be a chronic condition. She asks the nurse what chronic condition means. What would be the nurses best response?
    A) Chronic conditions are defined as health problems that require management of several months or longer.
    B) Chronic conditions are diseases that come and go in a relatively predictable cycle.
    C) Chronic conditions are medical conditions that culminate in disabilities that require hospitalization.
    D) Chronic conditions are those that require short-term management in extended-care facilities.
A

A) Chronic conditions are defined as health problems that require management of several months or longer.

Feedback:
Chronic conditions are often defined as medical conditions or health problems with associated symptoms or disabilities that require long-term management (3 months or longer). Chronic diseases are usually managed in the home environment. They are not always cyclical or predictable.

124
Q

A nursing is planning the care of a patient with emphysema who will soon be discharged. What teaching should the nurse prioritize in the plan of care?
A. Taking prophylactic antibiotics as ordered
B. B. Adhering to the treatment regimen in order to cure the disease
C. Avoiding airplanes, buses, and other crowded public places
D. Setting realistic short-term and long-range goals

A

D. Setting realistic short-term and long-range goals

Feedback:
A major area of teaching involves setting and accepting realistic short-term and long-range goals. Emphysema is not considered curable and antibiotics are not used on a preventative basis. The patient does not normally need to avoid public places.

125
Q

3.A patient with type 2 diabetes achieves adequate glycemic control through diet and exercise. Upon being admitted to the hospital for a cholecystectomy, however, the patient has required insulin injections on two occasions. The nurse would identify what likely cause for this short-term change in treatment?
A) Alterations in bile metabolism and release have likely caused hyperglycemia.
B) Stress has likely caused an increase in the patients blood sugar levels.
C) The patient has likely overestimated her ability to control her diabetes using nonpharmacologic measures.
D) The patients volatile fluid balance surrounding surgery has likely caused unstable blood sugars.

A

B) Stress has likely caused an increase in the patients blood sugar levels.

Feedback:
During periods of physiologic stress, such as surgery, blood glucose levels tend to increase, because levels of stress hormones (epinephrine, norepinephrine, glucagon, cortisol, and growth hormone) increase. The patients need for insulin is unrelated to the action of bile, the patients overestimation of previous blood sugar control, or fluid imbalance.

126
Q
  1. A patient has been brought to the ED with multiple trauma after a motor vehicle accident. After immediate threats to life have been addressed, the nurse and trauma team should take what action?
    A) Perform a rapid physical assessment.
    B) Initiate health education.
    C) Perform diagnostic imaging.
    D) Establish the circumstances of the accident.
A

A) Perform a rapid physical assessment.

Feedback:
Once immediate threats to life have been corrected, a rapid physical examination is done to identify injuries and priorities of treatment. Health education is initiated later in the care process and diagnostic imaging would take place after a rapid physical assessment. It is not the care teams responsibility to determine the circumstances of the accident.

127
Q
  1. A patient is brought by friends to the ED after being involved in a motor vehicle accident. The patient sustained blunt trauma to the abdomen. What nursing action would be most appropriate for this patient?
    A) Ambulate the patient to expel flatus.
    B) Place the patient in a high Fowlers position.
    C) Immobilize the patient on a backboard.
    D) Place the patient in a left lateral position.
A

C) Immobilize the patient on a backboard.

Feedback:
When admitted for blunt trauma, patients must be immobilized until spinal injury is ruled out. Ambulation, side-lying, and upright positioning would be contraindicated until spinal injury is ruled out.

128
Q
  1. A patient with chronic open-angle glaucoma is being taught to self-administer pilocarpine. After the patient administers the pilocarpine, the patient states that her vision is blurred. Which nursing action is most appropriate?
    A. Holding the next dose and notifying the physician
    B. Treating the patient for an allergic reaction
    C. Suggesting that the patient put on her glasses
    D. Explaining that this is an expected adverse effect
A

D. Explaining that this is an expected adverse effect

129
Q
  1. The nurse should recognize the greatest risk for the development of blindness in which of the following patients?
    A) A 58-year-old Caucasian woman with macular degeneration
    B. A 28-year-old Caucasian man with astigmatism
    C. A 58-year-old African American woman with hyperopia
    D.A 28-year-old African American man with myopia
A

A) A 58-year-old Caucasian woman with macular degeneration

Feedback:
The most common causes of blindness and visual impairment among adults 40 years of age or older are diabetic retinopathy, macular degeneration, glaucoma, and cataracts. The 58-year-old Caucasian woman with macular degeneration has the greatest risk for the development of blindness related to her age and the presence of macular degeneration. Individuals with hyperopia, astigmatism, and myopia are not in a risk category for blindness.

130
Q
  1. A 6-year-old child is brought to the pediatric clinic for the assessment of redness and discharge from the eye and is diagnosed with viral conjunctivitis. What is the most important information to discuss with the parents and child?
    A. Handwashing can prevent the spread of the disease to others.
    B. The importance of compliance with antibiotic therapy
    C. Signs and symptoms of complications, such as meningitis and septicemia
    D. The likely need for surgery to prevent scarring of the conjunctiva
A

A. Handwashing can prevent the spread of the disease to others.

Feedback:
The nurse must inform the parents and child that viral conjunctivitis is highly contagious and instructions should emphasize the importance of handwashing and avoiding sharing towels, face cloths, and eye drops. Viral conjunctivitis is not responsive to any treatment, including antibiotic therapy. Patients with gonococcal conjunctivitis are at risk for meningitis and generalized septicemia; these conditions do not apply to viral conjunctivitis. Surgery to prevent scarring of the conjunctiva is not associated with viral conjunctivitis.

131
Q
11) A nurse is caring for a patient diagnosed with Mnires disease. While completing a neurologic examination on the patient, the nurse assesses cranial nerve VIII. The nurse would be correct in identifying the function of this nerve as what? 
A)  Movement of the tongue 
B)  Visual acuity 
C)  Sense of smell 
D)  Hearing and equilibrium
A

D) Hearing and equilibrium

Feedback:
Cranial nerve VIII (acoustic) is responsible for hearing and equilibrium. Cranial nerve XII (hypoglossal) is responsible for movement of the tongue. Cranial nerve II (optic) is responsible for visual acuity and visual fields. Cranial nerve I (olfactory) functions in sense of smell.

132
Q
The nurse is providing discharge education for a patient with a new diagnosis of Mnires disease. What food should the patient be instructed to limit or avoid?
A)  Sweet pickles
B)  Frozen yogurt
C)  Shellfish
D)  Red meat
A

A) Sweet pickles

Feedback:
The patient with Mnires disease should avoid foods high in salt and/or sugar; sweet pickles are high in both. Milk products are not contraindicated. Any type of meat, fish, or poultry is permitted, with the exception of canned or pickled varieties. In general, the patient with Mnires disease should avoid or limit canned and processed foods.
LOW SODIUM DIET

133
Q

The nurse and a colleague are performing the Epley maneuver with a patient who has a diagnosis of benign paroxysmal positional vertigo. The nurses should begin this maneuver by performing what action?
A) Placing the patient in a prone position
B) Assisting the patient into a sitting position
C) Instilling 15 mL of warm normal saline into one of the patients ears
D) Assessing the patients baseline hearing by performing the whisper test

A

B) Assisting the patient into a sitting position

134
Q
The nurse is assessing a patient with multiple sclerosis who is demonstrating involuntary, rhythmic eye movements. What term will the nurse use when documenting these eye movements?
A)  Vertigo
B)  Tinnitus
C)  Nystagmus
D)  Astigmatism
A

C) Nystagmus

Feedback:
Vertigo is an illusion of movement where the individual or the surroundings are sensed as moving. Tinnitus refers to a subjective perception of sound with internal origin. Nystagmus refers to involuntary rhythmic eye movement. Astigmatism is a defect is visual acuity.

135
Q
15. A patient is complaining of pain in her casted leg. The nurse has administered analgesics and elevated the limb. Thirty minutes after administering the analgesics, the patient states the pain is unrelieved. The nurse should identify the warning signs of what complication?
A)  Subcutaneous emphysema
B)  Skin breakdown
C) Compartment syndrome 
D) Disuse syndrome
A

C) Compartment syndrome

FEEDBACK: Compartment syndrome may manifest as unrelenting, uncontrollable pain. This presentation of pain is not suggestive of disuse syndrome or skin breakdown. Subcutaneous emphysema is not a complication of casting.

136
Q
  1. A patient with a fractured femur is in balanced suspension traction. The patient needs to be repositioned toward the head of the bed. During repositioning, what should the nurse do?
    A) Place slight additional tension on the traction cords.
    B) Release the weights and replace them immediately after positioning.
    C) Reposition the bed instead of repositioning the patient.
    D) Maintain consistent traction tension while repositioning.
A

D) Maintain consistent traction tension while repositioning.

137
Q
  1. A patient with a right tibial fracture is being discharged home after having a cast applied. What instruction should the nurse provide in relationship to the patients cast care?
    A) Cover the cast with a blanket until the cast dries.
    B) Keep your right leg elevated above heart level.
    C) Use a clean object to scratch itches inside the cast.
    D) A foul smell from the cast is normal after the first few days.
A

B) Keep your right leg elevated above heart level.

Feedback:
The leg should be elevated to promote venous return and prevent edema. The cast shouldnt be covered while drying because this will cause heat buildup and prevent air circulation. No foreign object should be inserted inside the cast because of the risk of cutting the skin and causing an infection. A foul smell from a cast is never normal and may indicate an infection.

138
Q
  1. Which of the following patients should the nurse recognize as being at the highest risk for the development of osteomyelitis?
    A) A middle-age adult who takes ibuprofen daily for rheumatoid arthritis
    B) An elderly patient with an infected pressure ulcer in the sacral area
    C) A 17-year-old football player who had orthopedic surgery 6 weeks prior
    D) An infant diagnosed with jaundice
A

B) An elderly patient with an infected pressure ulcer in the sacral area

Feedback:
Patients who are at high risk of osteomyelitis include those who are poorly nourished, elderly, and obese. The elderly patient with an infected sacral pressure ulcer is at the greatest risk for the development of osteomyelitis, as this patient has two risk factors: age and the presence of a soft-tissue infection that has the potential to extend into the bone. The patient with rheumatoid arthritis has one risk factor and the infant with jaundice has no identifiable risk factors. The patient 6 weeks postsurgery is beyond the usual window of time for the development of a postoperative surgical wound infection.

139
Q
  1. A nurse is collaborating with the physical therapist to plan the care of a patient with osteomyelitis. What principle should guide the management of activity and mobility in this patient?
    A. Stress on the weakened bone must be avoided.
    B. Increased heart rate enhances perfusion and bone healing.
    C. Bed rest results in improved outcomes in patients with osteomyelitis. Maintenance of baseline
    D. ADLs is the primary goal during osteomyelitis treatment.
A

A. Stress on the weakened bone must be avoided.

140
Q
19. The nurse is assessing a patient for dietary factors that may influence her risk for osteoporosis. The nurse should question the patient about her intake of what nutrients? Select all that apply.
A)  Calcium
B)  Simple carbohydrates
C)  Vitamin D
D)  Protein
E)  Soluble fiber
A

A) Calcium

C) Vitamin D

141
Q
  1. A nursing educator is reviewing the risk factors for osteoporosis with a group of recent graduates. What risk factor of the following should the educator describe?
    A) Recurrent infections and prolonged use of NSAIDs
    B) High alcohol intake and low body mass index
    C) Small frame, female gender, and Caucasian ethnicity
    D) Male gender, diabetes, and high protein intake
A

C) Small frame, female gender, and Caucasian ethnicity

Feedback:
Small-framed, nonobese Caucasian women are at greatest risk for osteoporosis. Diabetes, high protein intake, alcohol use, and infections are not among the most salient risk factors for osteoporosis.

142
Q
  1. A nurse is providing a class on osteoporosis at the local seniors center. Which of the following statements related to osteoporosis is most accurate?
    A) Osteoporosis is categorized as a disease of the elderly.
    B) A nonmodifiable risk factor for osteoporosis is a persons level of activity.
    C) Secondary osteoporosis occurs in women after menopause.
    D) Slow discontinuation of corticosteroid therapy can halt the progression of the osteoporosis.
A

D) Slow discontinuation of corticosteroid therapy can halt the progression of the osteoporosis.

143
Q
19. An elderly female with osteoporosis has been hospitalized. Prior to discharge, when teaching the patient, the nurse should include information about which major complication of osteoporosis?
A)  Bone fracture
B)  Loss of estrogen
C. Negative calcium balance 
D.Dowagers hump
A

A) Bone fracture

144
Q
  1. A patient has been experiencing disconcerting GI symptoms that have been worsening in severity. Following medical assessment, the patient has been diagnosed with lactose intolerance. The nurse should recognize an increased need for what form of health promotion?
    A) Annual screening colonoscopies
    B) Adherence to recommended immunization schedules
    C) Regular blood pressure monitoring
    D) Frequent screening for osteoporosis
A

D) Frequent screening for osteoporosis

Feedback:
Persons with lactose intolerance often experience hypocalcemia and a consequent risk of osteoporosis related to malabsorption of calcium. Lactose intolerance does not create an increased need for screening for colorectal cancer, immunizations, or blood pressure monitoring.

145
Q
  1. The nurse is caring for a male patient whose diagnosis of bone cancer is causing severe and increasing pain. Before introducing nonpharmacological pain control interventions into the patients plan of care, the nurse should teach the patient which of the following?
    A) Nonpharmacological interventions must be provided by individuals other than members of the healthcare team.
    B) These interventions will not directly reduce pain, but will refocus him on positive stimuli.
    C) These interventions carry similar risks of adverse effects as analgesics.
    D) Reducing his use of analgesics is not the purpose of these interventions.
A

D) Reducing his use of analgesics is not the purpose of these interventions.

Feedback:
Patients who have been taking analgesic agents may mistakenly assume that clinicians suggest a nonpharmacolgical method to reduce the use or dose of analgesic agents. Nonpharmacological interventions indeed reduce pain and their use is not limited to practitioners outside the healthcare team. In general, adverse effects are minimal.

146
Q
  1. A medical nurse is providing end-of-life care for a patient with metastatic bone cancer. The nurse notes that the patient has been receiving oral analgesics for her pain with adequate effect, but is now having difficulty swallowing the medication. What should the nurse do?
    A) Request the physician to order analgesics by an alternative route.
    B) Crush the medication in order to aid swallowing and absorption.
    C) Administer the patients medication with the meal tray.
    D) Administer the medication rectally.
A

A) Request the physician to order analgesics by an alternative route.

147
Q
  1. The nurse is administering total parenteral nutrition (TPN) to a client who underwent surgery for gastric cancer. Which of the nurses assessments most directly addresses a major complication of TPN?
    A. Checking the patients capillary blood glucose levels regularly
    B. Having the patient frequently rate his or her hunger on a 10-point scale
    C. Measuring the patientsheart rhythm at least every 6 hours
    D. Monitoring the patients level of consciousness each shift
A

A. Checking the patients capillary blood glucose levels regularly

FEEDBACK:
The solution, used as a base for most TPN, consists of a high dextrose concentration and may raise blood glucose levels significantly, resulting in hyperglycemia. This is a more salient threat than hunger, though this should be addressed. Dysrhythmias and decreased LOC are not among the most common complications.

148
Q

You are caring for a patient with a diagnosis of pancreatitis. The patient was admitted from a homeless shelter and is a vague historian. The patient appears malnourished and on day 3 of the patients admission total parenteral nutrition (TPN) has been started. Why would you know to start the infusion of TPN slowly?
A) Patients receiving TPN are at risk for hypercalcemia if calories are started too rapidly.
B) Malnourished patients receiving parenteral nutrition are at risk for hypophosphatemia if calories are started too aggressively.
C) Malnourished patients who receive fluids too rapidly are at risk for hypernatremia.
D) Patients receiving TPN need a slow initiation of treatment in order to allow digestive enzymes to accumulate

A

B) Malnourished patients receiving parenteral nutrition are at risk for hypophosphatemia if calories are started too aggressively.

Feedback:
The nurse identifies patients who are at risk for hypophosphatemia and monitors them. Because malnourished patients receiving parenteral nutrition are at risk when calories are introduced too aggressively, preventive measures involve gradually introducing the solution to avoid rapid shifts of phosphorus into the cells. Patients receiving TPN are not at risk for hypercalcemia or hypernatremia if calories or fluids are started to rapidly. Digestive enzymes are not a relevant consideration.

149
Q

A patient with a diagnosis of gastric cancer has been unable to tolerate oral food and fluid intake and her tumor location precludes the use of enteral feeding. What intervention should the nurse identify as best meeting this patients nutritional needs?
A) Administration of parenteral feeds via a peripheral IV
B) TPN administered via a peripherally inserted central catheter
C) Insertion of an NG tube for administration of feeds
D) Maintaining NPO status and IV hydration until treatment completion

A

B) TPN administered via a peripherally inserted central catheter

Feedback:
If malabsorption is severe, or the cancer involves the upper GI tract, parenteral nutrition may be necessary. TPN is administered by way of a central line, not a peripheral IV. An NG would be contraindicated for this patient. Long-term NPO status would result in malnutrition.

150
Q
25. A staff educator is reviewing the causes of gastroesophageal reflux disease (GERD) with new staff nurses. What area of the GI tract should the educator identify as the cause of reduced pressure associated with GERD?
A)  Pyloric sphincter
B)  Lower esophageal sphincter
C)  Hypopharyngeal sphincter
D)  Upper esophageal sphincter
A

B) Lower esophageal sphincter

151
Q
  1. The nurses comprehensive assessment of a patient includes inspection for signs of oral cancer. What assessment finding is most characteristic of oral cancer in its early stages?
    A) Dull pain radiating to the ears and teeth
    B) Presence of a painless sore with raised edges
    C) Areas of tenderness that make chewing difficult
    D) Diffuse inflammation of the buccal mucosa
A

B) Presence of a painless sore with raised edges

Feedback:
Malignant lesions of the oral cavity are most often painless lumps or sores with raised borders. Because they do not bother the patient, delay in seeking treatment occurs frequently, and negatively affects prognosis. Dull pain radiating to the ears and teeth is characteristic of malocclusion. Inflammation of the buccal mucosa causes discomfort and often occurs as a side effect of chemotherapy. Tenderness resulting in pain on chewing may be associated with gingivitis, abscess, irritation from dentures, and other causes. Pain related to oral cancer is a late symptom.

152
Q
  1. A nurse who provides care in an ambulatory clinic integrates basic cancer screening into admission assessments. What patient most likely faces the highest immediate risk of oral cancer?
    A) A 65-year-old man with alcoholism who smokes
    B) A 45-year-old woman who has type 1 diabetes and who wears dentures
    C) A 32-year-old man who is obese and uses smokeless tobacco
    D) A 57-year-old man with GERD and dental caries
A

A) A 65-year-old man with alcoholism who smokes

153
Q
  1. A nurse is caring for a patient who just has been diagnosed with a peptic ulcer. When teaching the patient about his new diagnosis, how should the nurse best describe a peptic ulcer?
    A) Inflammation of the lining of the stomach
    B) Erosion of the lining of the stomach or intestine
    C) Bleeding from the mucosa in the stomach
    D) Viral invasion of the stomach wall
A

B) Erosion of the lining of the stomach or intestine

Feedback:
A peptic ulcer is erosion of the lining of the stomach or intestine. Peptic ulcers are often accompanied by bleeding and inflammation, but these are not the definitive characteristics.

154
Q
  1. A nurse is assessing a patient who has peptic ulcer disease. The patient requests more information about the typical causes of Helicobacter pylori infection. What would it be appropriate for the nurse to instruct the patient?
    A) Most affected patients acquired the infection during international travel.
    B) Infection typically occurs due to ingestion of contaminated food and water.
    C) Many people possess genetic factors causing a predisposition to H. pylori infection.
    D)The H. pylori microorganism is endemic in warm, moist climates.
A

B) Infection typically occurs due to ingestion of contaminated food and water.

Feedback:
Most peptic ulcers result from infection with the gram-negative bacteria H. pylori, which may be acquired through ingestion of food and water. The organism is endemic to all areas of the United States. Genetic factors have not been identified.

155
Q
32, A nurse who provides care in a campus medical clinic is performing an assessment of a 21-year-old student who has presented for care. After assessment, the nurse determines that the patient has a BMI of 45. What does this indicate?
A)  The patient is a normal weight.
B)  The patient is extremely obese.
C)  The patient is overweight.
D)  The patient is mildly obese.
A

B) The patient is extremely obese.

Feedback:
Individuals who have a BMI between 25 and 29.9 are considered overweight. Obesity is defined as a BMI of greater than 30 (WHO, 2011). A BMI of 45 would indicate extreme obesity.

156
Q
  1. A nurse is assessing a patient for risk factors known to contribute to osteoarthritis. What assessment finding would the nurse interpret as a risk factor?
    A) The patient has a 30 pack-year smoking history.
    B) The patients body mass index is 34 (obese).
    C) The patient has primary hypertension.
    D) The patient is 58 years old.
A

B) The patients body mass index is 34 (obese).

Feedback:
Risk factors for osteoarthritis include obesity and previous joint damage. Risk factors of OA do not include smoking or hypertension. Incidence increases with age, but a patient who is 58 would not yet face a significantly heightened risk.

157
Q
  1. A nurse is performing the admission assessment of a patient whose high body mass index (BMI) corresponds to class III obesity. In order to ensure empathic and patient-centered care, the nurse should do which of the following?
    A) Examine ones own attitudes towards obesity in general and the patient in particular.
    B) Dialogue with the patient about the lifestyle and psychosocial factors that resulted in obesity.
    C) Describe ones own struggles with weight gain and weight loss to the patient.
    D) Elicit the patients short-term and long-term goals for weight loss.
A

A) Examine ones own attitudes towards obesity in general and the patient in particular.

158
Q
  1. Which statement by the nurse is most likely to help a morbidly obese 22-year-old man in losing weight on a 1000-calorie diet?
    A) It will be necessary to change lifestyle habits permanently to maintain weight loss.
    B) You will decrease your risk for future health problems such as diabetes by losing weight now.
    C) You are likely to notice changes in how you feel with just a few weeks of diet and exercise.
    D) Most of the weight that you lose during the first weeks of dieting is water weight rather than fat.
A

C) You are likely to notice changes in how you feel with just a few weeks of diet and exercise.

159
Q
  1. In your role as a school nurse, you are working with a female high school junior whose BMI is 31. When planning this girls care, you should identify what goal?
    A) Continuation of current diet and activity level
    B) Increase in exercise and reduction in calorie intake
    C) Possible referral to an eating disorder clinic
    D) Increase in daily calorie intake
A

B) Increase in exercise and reduction in calorie intake

160
Q

34) The nurse is screening clients at a health fair. Which client is at highest risk for the development of colon cancer?
A) Older white client with irritable bowel syndrome
B) Middle-aged African-American client who smokes cigars
C) Middle-aged Asian client who travels and eats out frequently
D) Older American Indian client taking hormone replacement therapy

A

B) Middle-aged African-American client who smokes cigars

Feedback:
Colon cancer is more prevalent among African Americans and smokers. Irritable bowel syndrome, travel, and hormone replacement therapy do not increase the risk for colon cancer.

161
Q
34) A nurse is teaching a group of adults about screening and prevention of colorectal cancer. The nurse should describe which of the following as the most common sign of possible colon cancer? 
A)  Development of new hemorrhoids 
B)  Abdominal bloating and flank pain 
C)  Unexplained weight gain 
D)  Change in bowel habits
A

D) Change in bowel habits

Feedback: The most common presenting symptom associated with colorectal cancer is a change in bowel habits. The passage of blood is the second most common symptom. Symptoms may also include unexplained anemia, anorexia, weight loss, and fatigue. Hemorrhoids and bloating are atypical.

162
Q

35) A client informs the nurse that he has been having abdominal pain that is relieved when having a bowel movement. The client states that the physician told him he has irritable bowel syndrome. What does the nurse recognize as characteristic of this disorder?

  1. Blood and mucus in the stool
  2. Chronic constipation with sporadic bouts of diarrhea
  3. Weight loss due to malabsorption
  4. Client is awakened from sleep due to abdominal pain.
A
  1. Chronic constipation with sporadic bouts of diarrhea

Feedback:
Most clients with irritable bowel syndrome (IBS) describe having chronic constipation with sporadic bouts of diarrhea. Some report the opposite pattern, although less commonly. Most clients experience various degrees of abdominal pain that defecation may relieve. Weight usually remains stable, indicating that when diarrhea occurs,

163
Q

37) The nurse is caring for a patient who is undergoing diagnostic testing for suspected malabsorption. When taking this patients health history and performing the physical assessment, the nurse should recognize what finding as most consistent with this diagnosis?
A) Recurrent constipation coupled with weight loss
B) Foul-smelling diarrhea that contains fat
C) Fever accompanied by a rigid, tender abdomen
D) Bloody bowel movements accompanied by fecal incontinence

A

B) Foul-smelling diarrhea that contains fat

Feedback:
The hallmarks of malabsorption syndrome from any cause are diarrhea or frequent, loose, bulky, foul- smelling stools that have increased fat content and are often grayish (steatorrhea). Constipation and bloody bowel movements are not suggestive of malabsorption syndromes. Fever and a tender, rigid abdomen are associated with peritonitis.

164
Q
37) Diagnostic imaging and physical assessment have revealed that a patient with peptic ulcer disease has suffered a perforated ulcer. The nurse recognizes that emergency interventions must be performed as soon as possible in order to prevent the development of what complication? 
A)  Peritonitis 
B)  Gastritis 
C)  Gastroesophageal reflux 
D)  Acute pancreatitis
A

A) Peritonitis

Feedback:
Perforation is the erosion of the ulcer through the gastric serosa into the peritoneal cavity without warning. Chemical peritonitis develops within a few hours of perforation and is followed by bacterial peritonitis. Gastritis, reflux, and pancreatitis are not acute complications of a perforated ulcer.

165
Q

38) An elderly patient has presented to the clinic with a new diagnosis of osteoarthritis. The patients daughter is accompanying him and you have explained why the incidence of chronic diseases tends to increase with age. What rationale for this phenomenon should you describe?
A) With age, biologic changes reduce the efficiency of body systems.
B) Older adults often have less support and care from their family, resulting in illness.
C) There is an increased morbidity of peers in this age group, and this leads to the older adults desire to also assume the sick role.
D) Chronic illnesses are diagnosed more often in older adults because they have more contact with the health care system.

A

A) With age, biologic changes reduce the efficiency of body systems.

Feedback:
Causes of the increasing number of people with chronic conditions include the following: longer lifespans because of advances in technology and pharmacology, improved nutrition, safer working conditions, and greater access (for some people) to health care. Also, biologic conditions change in the aged population. These changes reduce the efficiency of the bodys systems. Older adults usually have more support and care from their family members. Assuming the sick role can be a desire in any age group, not just the elderly.

166
Q

39) A nurse is teaching an adult female patient about the risk factors for hypertension. What should the nurse explain as risk factors for primary hypertension?
A) Obesity and high intake of sodium and saturated fat
B) Diabetes and use of oral contraceptives
C) Metabolic syndrome and smoking
D) Renal disease and coarctation of the aorta

A

A) Obesity and high intake of sodium and saturated fat

Feedback:
Obesity, stress, high intake of sodium or saturated fat, and family history are all risk factors for primary hypertension. Diabetes and oral contraceptives are risk factors for secondary hypertension. Metabolic syndrome, renal disease, and coarctation of the aorta are causes of secondary hypertension.

167
Q

39) The nurse is performing stroke risk screenings at a hospital open house. The nurse has identified four patients who might be at risk for a stroke. Which patient is likely at the highest risk for a hemorrhagic stroke?
A) White female, age 60, with history of excessive alcohol intake
B) White male, age 60, with history of uncontrolled hypertension
C) Black male, age 60, with history of diabetes
D) Black male, age 50, with history of smoking

A

B) White male, age 60, with history of uncontrolled hypertension

Feedback:
Uncontrolled hypertension is the primary cause of a hemorrhagic stroke. Control of hypertension, especially in individuals over 55 years of age, clearly reduces the risk for hemorrhagic stroke. Additional risk factors are increased age, male gender, and excessive alcohol intake. Another high-risk group includes African Americans, where the incidence of first stroke is almost twice that as in Caucasians.

168
Q
40. A nurse is performing blood pressure screenings at a local health fair. While obtaining subjective assessment data from a patient with hypertension, the nurse learns that the patient has a family history of hypertension and she herself has high cholesterol and lipid levels. The patient says she smokes one pack of cigarettes daily and drinks about a pack of beer every day. The nurse notes what nonmodifiable risk factor for hypertension?
A)  Hyperlipidemia
B)  Excessive alcohol intake
C)  A family history of hypertension
D)  Closer adherence to medical regimen
A

C) A family history of hypertension

Feedback:
Unlike cholesterol levels, alcohol intake and adherence to treatment, family history is not modifiable.

169
Q
40. A community health nurse is giving an educational presentation about stroke and heart disease at the local senior citizens center. What nonmodifiable risk factor for stroke should the nurse cite? 
A.	Female gender
B.	Asian American race
C.	Advanced age
D.	Smoking
A

C. Advanced age

Feedback: Advanced age, male gender, and race are well-known nonmodifiable risk factors for stroke. High-risk groups include people older than 55 years of age; the incidence of stroke more than doubles in each successive decade. Men have a higher rate of stroke than that of women. Another high-risk group is African Americans; the incidence of first stroke in African Americans is almost twice that as in Caucasian Americans; Asian American race is not a risk factor. Smoking is a modifiable risk.

170
Q

The nurse is collaborating with the dietitian and a patient with hypertension to plan dietary modifications. These modifications should include which of the following?
A) Reduced intake of protein and carbohydrates
B) Increased intake of calcium and vitamin D
C) Reduced intake of fat and sodium
D) Increased intake of potassium, vitamin B12 and vitamin D

A

C) Reduced intake of fat and sodium

Feedback:
Lifestyle modifications usually include restricting sodium and fat intake, increasing intake of fruits and vegetables, and implementing regular physical activity. There is no need to increase calcium, potassium, and vitamin intake. Calorie restriction may be required for some patients, but a specific eduction in protein and carbohydrates is not normally indicated.

171
Q

A patient has been diagnosed as being prehypertensive. What should the nurse encourage this patient to do to aid in preventing a progression to a hypertensive state? A) Avoid excessive potassium intake.
B) Exercise on a regular basis.
C) Eat less protein and more vegetables.
D) Limit morning activity.

A

B) Exercise on a regular basis

Feedback: To prevent or delay progression to hypertension and reduce risk, JNC 7 urged health care providers to encourage people with blood pressures in the prehypertension category to begin lifestyle modifications, such as nutritional changes and exercise. There is no need for patients to limit their activity in the morning or to avoid potassium and protein intake.

172
Q

A 55-year-old man has been newly diagnosed with acute pancreatitis and admitted to the acute medical unit. How should the nurse most likely explain the pathophysiology of this patient’s health problem?
A) “Toxins have accumulated and inflamed your pancreas.”
B) “Bacteria likely migrated from your intestines and became lodged in your pancreas.”
C) “A virus that was likely already present in your body has begun to attack your pancreatic cells.”
D) “The enzymes that your pancreas produces have damaged the pancreas itself.”

A

D) “The enzymes that your pancreas produces have damaged the pancreas itself.”

Feedback:
Although the mechanisms causing pancreatitis are unknown, pancreatitis is commonly described as the autodigestion of the pancreas. Less commonly, toxic substances and microorganisms are implicated as the cause of pancreatitis.

173
Q

A 37-year-old male patient presents at the emergency department (ED) complaining of nausea and vomiting and severe abdominal pain. The patient’s abdomen is rigid, and there is bruising to the patient’s flank. The patient’s wife states that he was on a drinking binge for the past 2 days. The ED nurse should assist in assessing the patient for what health problem?
A) Severe pancreatitis with possible peritonitis
B) Acute cholecystitis
C) Chronic pancreatitis
D) Acute appendicitis with possible perforation

A

A) Severe pancreatitis with possible peritonitis

Feedback:
Severe abdominal pain is the major symptom of pancreatitis that causes the patient to seek medical care. Pain in pancreatitis is accompanied by nausea and vomiting that does not relieve the pain or nausea. Abdominal guarding is present and a rigid or board-like abdomen may be a sign of peritonitis. Ecchymosis (bruising) to the flank or around the umbilicus may indicate severe peritonitis. Pain generally occurs 24 to 48 hours after a heavy meal or alcohol ingestion. The link with alcohol intake makes pancreatitis a more likely possibility than appendicitis or cholecystitis.

174
Q

A newly admitted patient with type 1 diabetes asks the nurse what caused her diabetes. When the nurse is explaining to the patient the etiology of type 1 diabetes, what process should the nurse describe?
. A) The tissues in your body are resistant to the action of insulin, making the glucose levels in your blood increase.
. B) Damage to your pancreas causes an increase in the amount of glucose that it releases, and there is not enough insulin to control it.
. C) The amount of glucose that your body makes overwhelms your pancreas and decreases your production of insulin.
. D) Destruction of special cells in the pancreas causes a decrease in insulin production. Glucose levels rise because insulin normally breaks it down.

A

D) Destruction of special cells in the pancreas causes a decrease in insulin production. Glucose levels rise because insulin normally breaks it down.

Feedback:
Type 1 diabetes is characterized by the destruction of pancreatic beta cells, resulting in decreased insulin production, unchecked glucose production by the liver, and fasting hyperglycemia. Also, glucose derived from food cannot be stored in the liver and remains circulating in the blood, which leads to postprandial hyperglycemia. Type 2 diabetes involves insulin resistance and impaired insulin secretion. The body does not make glucose

175
Q

A client is scheduled for a paracentesis. Which activity does the nurse delegate to the unlicensed assistive personnel?

a. Have the client sign the informed consent form.
b. Assist the client to void before the procedure.
c. Help the client lie flat in bed, on the right side.
d. Get the client into a chair after the procedure.

A

b. Assist the client to void before the procedure.

Feedback: For safety, the client should void just before a paracentesis. The nurse or the provider should have the client sign the consent form. The proper position for a paracentesis is sitting upright in bed or, alternatively, sitting on the side of the bed and leaning over the bedside table. The client will be on bedrest after the procedure.

176
Q

A client just had a paracentesis. Which nursing intervention is a priority for this client?

a. Monitor urine output.
b. Maintain bedrest as per protocol
c. Position the client flat in bed.
d. Secure the trocar to the abdomen with tape.

A

b. Maintain bedrest as per protocol

Feedback:
After a paracentesis, the client should remain on bedrest with the head of the bed elevated. A client with liver dysfunction is at risk for bleeding, and bedrest decreases this risk. Clients with liver dysfunction must have intake and output monitored, but this is not the priority after this procedure. A drain may be placed for short-term therapy in some clients.

177
Q

A client had a paracentesis 1 hour ago. Which assessment finding requires action by the nurse?

a. Urine output of 20 mL/hr
b. Systolic blood pressure increase of 10 mm Hg
c. Respiratory rate drop from 18 to 14
d. A 3-pound drop in weight

A

a. Urine output of 20 mL/hr

Feedback: Rapid removal of ascitic fluid causes decreased abdominal pressure, which can contribute to hypovolemia. This can be manifested by a decrease in urine output to below 30 mL/hr. The nurse would expect the client’s weight to drop as fluid is removed. A decrease in respiratory rate indicates that breathing has been made easier by the procedure. A slight increase in systolic blood pressure is insignificant.

178
Q

A nurse is amending a patients plan of care in light of the fact that the patient has recently developed ascites. What should the nurse include in this patients care plan?
A) Mobilization with assistance at least 4 times daily
B) Administration of beta-adrenergic blockers as ordered
C) Vitamin B12 injections as ordered
D) Administration of diuretics as ordered

A

D) Administration of diuretics as ordered

Feedback:
Use of diuretics along with sodium restriction is successful in 90% of patients with ascites. Beta- blockers are not used to treat ascites and bed rest is often more beneficial than increased mobility. Vitamin B12 injections are not necessary.

179
Q

A client has been diagnosed with hepatitis A. The nurse evaluates that teaching regarding the disease is understood when the client makes which statement?

a. “Some medications have been known to induce hepatitis A.”
b. “I may have been exposed when we ate shrimp last weekend.”
c. “I may have been infected through a recent blood transfusion.”
d. “My infection with Epstein-Barr virus can co-infect me with hepatitis A.”

A

b. “I may have been exposed when we ate shrimp last weekend.”

Feedback: The route of acquisition of hepatitis A infection is through close personal contact or ingestion of contaminated water or shellfish. The other statements are not accurate. Hepatitis B can be spread through sexual contact, needle sharing, needle sticks, blood transfusions, hemodialysis, acupuncture, and the maternal-fetal route. A person with multiple sexual partners has more opportunities to contract the infection.

180
Q

The nurse is administering total parenteral nutrition (TPN) to a Client who underwent surgery for gastric cancer. Which of the nurses assessments most directly addresses a major complication of TPN?
A) Checking the patients capillary blood glucose levels regularly
B) Having the patient frequently rate his or her hunger on a 10-point scale
C) Measuring the patients heart rhythm at least every 6 hours
D) Monitoring the patients level of consciousness each shift

A

A) Checking the patients capillary blood glucose levels regularly

Feedback: The solution, used as a base for most TPN, consists of a high dextrose concentration and may raise blood glucose levels significantly, resulting in hyperglycemia. This is a more salient threat than hunger, though this should be addressed. Dysrhythmias and decreased LOC are not among the most common

181
Q
A nurse is assessing a patient who has been diagnosed with cholecystitis, and is experiencing localized abdominal pain. When assessing the characteristics of the patients pain, the nurse should anticipate that it may radiate to what region? 
A) Left upper chest 
B) Inguinal region 
C) Neck or jaw 
D) Right shoulder
A

D) Right shoulder

Feedback: The patient may have biliary colic with excruciating upper right abdominal pain that radiates to the back or right shoulder. Pain from cholecystitis does not typically radiate to the left upper chest, inguinal area, neck,or jaw.

182
Q

A patient with a cholelithiasis has been scheduled for a laparoscopic cholecystectomy. Why is laparoscopic cholecystectomy preferred by surgeons over an open procedure?

A) Laparoscopic cholecystectomy poses fewer surgical risks than an open procedure.
B) Laparoscopic cholecystectomy can be performed in a clinic setting, while an open procedure requires an OR.
C) A laparoscopic approach allows for the removal of the entire gallbladder.
D) A laparoscopic approach can be performed under conscious sedation.

A

A) Laparoscopic cholecystectomy poses fewer surgical risks than an open procedure.

Feedback:
Open surgery has largely been replaced by laparoscopic cholecystectomy (removal of the gallbladder through a small incision through the umbilicus). As a result, surgical risks have decreased, along with the length of hospital stay and the long recovery period required after standard surgical cholecystectomy. Both approaches allow for removal of the entire gallbladder and must be performed under general anesthetic in an operating theater.

183
Q

A client was admitted with diabetic ketoacidosis (DKA). Which manifestations does the nurse monitor the client most closely for?

a. Shallow slow respirations and respiratory alkalosis
b. Decreased urine output and hyperkalemia
c. Tachycardia and orthostatic hypotension
d. Peripheral edema and dependent pulmonary crackles

A

c. Tachycardia and orthostatic hypotension

Feedback:
DKA leads to dehydration, which is manifested by tachycardia and orthostatic hypotension. Usually clients have Kussmaul respirations, which are fast and deep. Increased urinary output (polyuria) is severe. Because of diuresis and dehydration, peripheral edema and crackles do not occur.

184
Q

The nurse is caring for a critically ill client who has diabetic ketoacidosis (DKA). The nurse finds the following assessment data: blood pressure, 90/62; pulse, 120 beats/min; respirations, 28 breaths/min; urine output, 20 mL/1 hour per catheter; serum potassium, 2.6 mEq/L. The health care provider orders a 40 mEq potassium bolus and an increase in the IV flow rate. Which action by the nurse is most appropriate?

a. Give the potassium after increasing the IV flow rate.
b. Increase the IV rate; consult the provider about the potassium.
c. Increase the IV rate; hold the potassium for now.
d. Infuse the potassium first before increasing the IV flow rate.

A

b. Increase the IV rate; consult the provider about the potassium.

Feedback:
The client is acutely ill and is severely dehydrated and hypokalemic. The client requires more IV fluids and potassium. However, potassium should not be infused unless the urine output is at least 30 mL/hr. The nurse should first increase the IV rate, then consult with the provider about the potassium. The nurse should not just hold the potassium without consulting the provider because the client’s level is dangerously low.

185
Q

A client in the emergency department has been diagnosed with ketoacidosis. Which manifestation does the nurse correlate with this condition?

a. Increased rate and depth of respiration
b. Extremity tremors followed by seizure activity
c. Oral temperature of 102° F (38.9° C)
d. Severe orthostatic hypotension

A

a. Increased rate and depth of respiration

Feedback:
Ketoacidosis decreases the pH of the blood, stimulating the respiratory control areas of the brain to buffer the effects of increasing acidosis. The rate and depth of respiration are increased (Kussmaul respirations) in an attempt to excrete more acids by exhalation.

186
Q

A client has diabetic ketoacidosis and manifests Kussmaul respirations. What action by the nurse takes priority?

a. Administration of oxygen by mask or nasal cannula
b. Intravenous administration of 10% glucose
c. Implementation of seizure precautions
d. Administration of intravenous insulin

A

d. Administration of intravenous insulin

Feedback:
The rapid, deep respiratory efforts of Kussmaul respiration is the body’s attempt to reduce the acids produced by using fat rather than glucose for fuel. The client who is in ketoacidosis and who does not also have a respiratory impairment does not need additional oxygen. Only the administration of insulin will reduce this type of respiration by assisting glucose to move into cells and to be used for fuel instead of fat. Giving the client glucose would be contraindicated. The client does not require Seizure Precautions.

187
Q

A diabetic nurse is working for the summer at a camp for adolescents with diabetes. When providing information on the prevention and management of hypoglycemia, what action should the nurse promote?
A) Always carry a form of fast-acting sugar.
B) Perform exercise prior to eating whenever possible.
C) Eat a meal or snack every 8 hours.
D) Check blood sugar at least every 24 hours.

A

A) Always carry a form of fast-acting sugar.

Feedback:
The following teaching points should be included in information provided to the patient on how to prevent hypoglycemia: Always carry a form of fast-acting sugar, increase food prior to exercise, eat a meal or snack every 4 to 5 hours, and check blood sugar regularly.

188
Q

A student with diabetes tells the school nurse that he is feeling nervous and hungry. The nurse assesses the child and finds he has tachycardia and is diaphoretic with a blood glucose level of 50 mg/dL (2.8 mmol/L). What should the school nurse administer?
A) A combination of protein and carbohydrates, such as a small cup of yogurt
B) Two teaspoons of sugar dissolved in a cup of apple juice
C) Half of a cup of juice, followed by cheese and crackers
D) Half a sandwich with a protein-based filling

A

C) Half of a cup of juice, followed by cheese and crackers

Feedback:
Initial treatment for hypoglycemia is 15 g concentrated carbohydrate, such as two or three glucose tablets, 1 tube glucose gel, or 0.5 cup juice. After initial treatment, the nurse should follow with a snack including starch and protein, such as cheese and crackers, milk and crackers, or half of a sandwich. It is unnecessary to add sugar to juice, even it if is labeled as unsweetened juice, because the fruit sugar in juice contains enough simple carbohydrate to raise the blood glucose level and additional sugar may result in a sharp rise in blood sugar that will last for several hours.

189
Q

A school nurse is teaching a group of high school students about risk factors for diabetes. Which of the following actions has the greatest potential to reduce an individuals risk for developing diabetes?
A) Have blood glucose levels checked annually.
B) Stop using tobacco in any form.
C) Undergo eye examinations regularly.
D) Lose weight, if obese.

A

D) Lose weight, if obese.

Feedback:
Obesity is a major modifiable risk factor for diabetes. Smoking is not a direct risk factor for the disease. Eye examinations are necessary for persons who have been diagnosed with diabetes, but they do not screen for the disease or prevent it. Similarly, blood glucose checks do not prevent the diabetes.

190
Q

A client just diagnosed with acromegaly is scheduled for a hypophysectomy. Which statement made by the client indicates a need for clarification regarding this treatment?

a. “I will drink whenever I feel thirsty after surgery.”
b. “I’m glad no visible incision will result from this surgery.”
c. “I hope I can go back to wearing size 8 shoes instead of size 12.”
d. “I will wear slip-on shoes after surgery so I don’t have to bend over.”

A

c. “I hope I can go back to wearing size 8 shoes instead of size 12.”

Feedback:
Although removal of the tissue that is oversecreting hormones can relieve many symptoms of hyperpituitarism, skeletal changes and organ enlargement are not reversible. It will be appropriate for the client to drink as needed postoperatively and avoid bending over, reassured that the incision will not be visible.

191
Q

When taking the blood pressure of a client after a parathyroidectomy, the nurse notes that the client’s hand has gone into flexion contractions. Which laboratory result does the nurse correlate with this condition?

a. Serum potassium, 2.9 mEq/L
b. Serum potassium, 5.8 mEq/L
c. Serum sodium, 122 mEq/L
d. Serum calcium, 6.9 mg/dL

A

d. Serum calcium, 6.9 mg/dL

Feedback:
Hypocalcemia destabilizes excitable membranes and can lead to muscle twitches, spasms, and tetany. This effect of hypocalcemia is enhanced in the presence of tissue hypoxia. The flexion contractions (Trousseau’s sign) that occur during blood pressure measurement are indicative of hypocalcemia, not the other electrolyte imbalances, which include hypokalemia, hyperkalemia, and hyponatremia.

192
Q

A client is hospitalized with acute pancreatitis. The nursing assistant reports to the nurse that when a blood pressure cuff was applied, the client’s hand had a spasm. Which additional finding does the nurse correlate with this condition?

a. Serum calcium, 5.8 mg/dL
b. Serum sodium, 166 mEq/L
c. Serum creatinine, 0.9 mg/dL
d. Serum potassium, 4.2 mEq/dL

A

a. Serum calcium, 5.8 mg/dL

Feedback:
Spasm of the hand when a blood pressure cuff is applied (Trousseau’s sign) is indicative of hypocalcemia. The client’s calcium level is low. The sodium level is high, but that is not related to Trousseau’s sign. Creatinine and potassium levels are normal.

193
Q

The physician has ordered a fluid deprivation test for a patient suspected of having diabetes insipidus. During the test, the nurse should prioritize what assessments? A) Temperature and oxygen saturation
B) Heart rate and BP
C) Breath sounds and bowel sounds
D) Color, warmth, movement, and sensation of extremities

A

B) Heart rate and BP

Feedback: The fluid deprivation test is carried out by withholding fluids for 8 to 12 hours or until 3% to 5% of the body weight is lost. The patients condition needs to be monitored frequently during the test, and the test is terminated if tachycardia, excessive weight loss, or hypotension develops. Consequently, BP and heart rate monitoring are priorities over the other listed assessments.

194
Q

A patient has developed diabetes insipidus after having increased ICP following head trauma. What nursing assessment best addresses this complication?
A) Vigilant monitoring of fluid balance
B) Continuous BP monitoring
C) Serial arterial blood gases (ABGs)
D) Monitoring of the patients airway for patency

A

A) Vigilant monitoring of fluid balance

Feedback:
Diabetes insipidus requires fluid and electrolyte replacement, along with the administration of vasopressin, to replace and slow the urine output. Because of these alterations in fluid balance, careful monitoring is necessary. None of the other listed assessments directly addresses the major manifestations of diabetes insipidus.

195
Q

A patient with recurrent urinary tract infections has just undergone a cystoscopy and complains of slight hematuria during the first void after the procedure. What is the nurses most appropriate action?
A) Administer a STAT dose of vitamin K, as ordered.
B) Reassure the patient that this is not unexpected and then monitor the patient for further bleeding.
C) Promptly inform the physician of this assessment finding.
D) Position the patient supine and insert a Foley catheter, as ordered.

A

B) Reassure the patient that this is not unexpected and then monitor the patient for further bleeding.

Feedback:
Some burning on voiding, blood-tinged urine, and urinary frequency from trauma to the mucous membranes can be expected after cystoscopy. The nurse should explain this to the patient and ensure that the bleeding resolves. No clear need exists to report this finding and it does not warrant insertion of a Foley catheter or vitamin K administration.

196
Q

A patient is complaining of genitourinary pain shortly after returning to the unit from a scheduled cystoscopy. What intervention should the nurse perform?
A) Encourage mobilization.
B) Apply topical lidocaine to the patients meatus, as ordered.
C) Apply moist heat to the patients lower abdomen.
D) Apply an ice pack to the patients perineum.

A

C) Apply moist heat to the patients lower abdomen.

Feedback:
Following cystoscopy, moist heat to the lower abdomen and warm sitz baths are helpful in relieving pain and relaxing the muscles. Ice, lidocaine, and mobilization are not recommended interventions.

197
Q

A client has undergone diagnostic testing that involved the insertion of a lighted tube with a telescopic lens. The nurse identifies this test as which of the following?

a) Excretory urogram
b) Cystoscopy
c) Intravenous pyelography
d) Renal angiography

A

b) Cystoscopy

Feedback:
Cystoscopy is the visual examination of the inside of the bladder using an instrument called a cystoscope, a lighted tube with a telescopic lens. Renal angiography involves the passage of a catheter up the femoral artery into the aorta to the level of the renal vessels. Intravenous pyelography or excretory urography is a radiologic study that involves the use of a contrast medium to evaluate the kidneys’ ability to excrete it.

198
Q

The nurse is caring for a patient who is colonized with methicillin-resistant Staphylococcus aureus (MRSA). What infection control measure has the greatest potential to reduce transmission of MRSA and other nosocomial pathogens in a health care setting?
A) Using antibacterial soap when bathing patients with MRSA
B) Conducting culture surveys on a regularly scheduled basis
C) Performing hand hygiene before and after contact with every patient
D) Using aseptic housekeeping practices for environmental cleaning

A

C) Performing hand hygiene before and after contact with every patient

Feedback: Handwashing is the major infection control measure to reduce the risk of transmission of MRSA and other nosocomial pathogens. No convincing evidence exists to support that bathing patients with antibacterial soap is effective. Culture surveys can help establish the true prevalence of MRSA in a facility, but are used only to help implement where and when infection-control measures are needed. Hand hygiene is known to be more clinically important than house keeping.

199
Q
A 42-year-old woman comes to the clinic complaining of occasional urinary incontinence when she sneezes. The clinic nurse should recognize what type of incontinence? 
A)  Stress incontinence 
B)  Reflex incontinence 
C)  Overflow incontinence 
D)  Functional incontinence
A

A) Stress incontinence

Feedback:
Stress incontinence is the involuntary loss of urine through an intact urethra as a result of sudden increase in intra-abdominal pressure. Reflex incontinence is loss of urine due to hyperreflexia or involuntary urethral relaxation in the absence of normal sensations usually associated with voiding. Overflow incontinence is an involuntary urine loss associated with overdistension of the bladder. Functional incontinence refers to those instances in which the function of the lower urinary tract is intact, but other factors (outside the urinary system) make it difficult or impossible for the patient to

200
Q

The most common presenting objective symptoms of a urinary tract infection in older adults, especially in those with dementia, include?

a) Incontinence
b) Change in cognitive functioning
c) Back pain
d) Hematuria

A

b) Change in cognitive functioning

Feedback
The most common objective finding is a change in cognitive functioning, especially in those with dementia, because these patients usually exhibit even more profound cognitive changes with the onset of a UTI. Incontinence, hematuria, and back pain are not the most common presenting objective symptoms.

201
Q
A nurse is caring for a 73-year-old patient with a urethral obstruction related to prostatic enlargement. When planning this patients care, the nurse should be aware of the consequent risk of what complication? 
A)  Urinary tract infection 
B)  Enuresis 
C)  Polyuria 
D)  Proteinuria
A

A) Urinary tract infection

Feedback:
An obstruction of the bladder outlet, such as in advanced benign prostatic hyperplasia, results in abnormally high voiding pressure with a slow, prolonged flow of urine. The urine may remain in the bladder, which increases the potential of a urinary tract infection. Older male patients are at risk for prostatic enlargement, which causes urethral obstruction and can result in hydronephrosis, renal failure, and urinary tract infections.

202
Q
62) While taking a health history on a 20-year-old female patient, the nurse ascertains that this patient is taking miconazole (Monistat). The nurse is justified in presuming that this patient has what medical condition? 
A)  Bacterial vaginosis 
B)  Human papillomavirus (HPV) 
C)  Candidiasis 
D)  Toxic shock syndrome (TSS)
A

C) Candidiasis

Feedback:
Candidiasis is a fungal or yeast infection caused by strains of Candida. Miconazole (Monistat) is an antifungal medication used in the treatment of candidiasis. This agent is inserted into the vagina with an applicator at bedtime and may be applied to the vulvar area for pruritus. HPV, bacterial vaginosis, and TSS are not treated by Monistat.

203
Q

62) You are caring for an adult patient who has developed a mild oral yeast infection following chemotherapy. What actions should you encourage the patient to perform? Select all that apply.
A) Use a lip lubricant.
B) Scrub the tongue with a firm-bristled toothbrush.
C) Use dental floss every 24 hours.
D) Rinse the mouth with normal saline.
E) Eat spicy food to aid in eradicating the yeast.

A

A) Use a lip lubricant.
C) Use dental floss every 24 hours.
D) Rinse the mouth with normal saline.

Feedback:
Stomatitis is an inflammation of the oral cavity. The patient should be encouraged to brush the teeth with a soft toothbrush after meals, use dental floss every 24 hours, rinse with normal saline, and use a lip lubricant. Mouthwashes and hot foods should be avoided.

204
Q

63) A female patient has been prescribed a course of antibiotics for the treatment of a UTI. When providing health education for the patient, the nurse should address what topic?
A) The risk of developing a vaginal yeast infection as a consequent of antibiotic therapy
B) The need to expect a heavy menstrual period following the course of antibiotics
C) The risk of developing antibiotic resistance after the course of antibiotics
D) The need to undergo a series of three urine cultures after the antibiotics have been completed

A

A) The risk of developing a vaginal yeast infection as a consequent of antibiotic therapy

Feedback:
Yeast vaginitis occurs in as many as 25% of patients treated with antimicrobial agents that affect vaginal flora. Yeast vaginitis can cause more symptoms and be more difficult and costly to treat than the original UTI. Antibiotics do not affect menstrual periods and serial urine cultures are not normally necessary. Resistance is normally a result of failing to complete a prescribed course of antibiotics.

205
Q
77) A 93-year-old male patient with failure to thrive has begun exhibiting urinary incontinence. When choosing appropriate interventions, you know that various age-related factors can alter urinary elimination patterns in elderly patients. What is an example of these factors? 
A)	Decreased residual volume 
B)	Urethral stenosis
C)	Increased bladder capacity 
D)	Decreased muscle tone
A

D) Decreased muscle tone

Feedback:
Factors that alter elimination patterns in the older adult include decreased bladder capacity, decreased muscle tone, increased residual volumes, and delayed perception of elimination cues. The other noted phenomena are atypical.

206
Q

An 84-year-old patient has returned from the post-anesthetic care unit (PACU) following hip arthroplasty. The patient is oriented to name only. The patients family is very upset because, before having surgery, the patient had no cognitive deficits. The patient is subsequently diagnosed with postoperative delirium. What should the nurse explain to the patients family?
A. This problem is self-limiting and there is nothing to worry about.
B. Delirium involves a progressive decline in memory loss and overall cognitive function..
C. Delirium of this type is treatable and her cognition will return to previous levels.
D. This problem can be resolved by administering antidotes to the anesthetic that was used in surgery.

A

C. Delirium of this type is treatable and her cognition will return to previous levels.

Feedback:
Surgery is a common cause of delirium in older adults. Delirium differs from other types of dementia in that delirium begins with confusion and progresses to disorientation. It has symptoms that are reversible with treatment, and, with treatment, is short term in nature. It is patronizing and inaccurate to reassure the family that there is nothing to worry about. The problem is not treated by the administration of antidotes to anesthetic.

207
Q
78) The nurse is caring for an 88-year-old patient who is recovering from an ileac-femoral bypass graft. The patient is day 2 postoperative and has been mentally intact, as per baseline. When the nurse assesses the patient, it is clear that he is confused and has been experiencing disturbed sleep patterns and impaired psychomotor skills. What should the nurse suspect is the problem with the patient? 
A)  Postoperative delirium 
B)  Postoperative dementia 
C)  Senile dementia 
D)  Senile confusion
A

A) Postoperative delirium

Feedback:
Postoperative delirium, characterized by confusion, perceptual and cognitive deficits, altered attention levels, disturbed sleep patterns, and impaired psychomotor skills, is a significant problem for older adults. Dementia does not have a sudden onset. Senile confusion is not a recognized health problem.

208
Q

79) A critically ill patient is admitted to the ICU. The physician decides to use intra-arterial pressure monitoring. After this intervention is performed, what assessment should the nurse prioritize in the plan of care?
A) Fluctuations in core body temperature
B) Signs and symptoms of esophageal varices
C) Signs and symptoms of compartment syndrome
D) Perfusion distal to the insertion site

A

D) Perfusion distal to the insertion site

Feedback:
The radial artery is the usual site selected. However, placement of a catheter into the radial artery can further impede perfusion to an area that has poor circulation. As a result, the tissue distal to the cannulated artery can become ischemic or necrotic. Vigilant assessment is thus necessary. Alterations in temperature and the development of esophageal varices or compartment syndrome are not high risks.